Site Loader

Содержание

Электрический ток


Еще в 18 веке было доказано, что электрический ток способен оказывать сильное негативное влияние на человеческий организм. Но только спустя около века были сделаны первые описания электротравм, получаемых от воздействия постоянного тока (1863 г.) и переменного (1882 г.).

Что такое электротравма и электротравматизм?

Электротравма – повреждение человеческого организма электрическим током (электрической дугой).

Явление электротравматизма объясняется последовательностью следующих особенностей: в организме человека, случайно оказавшегося под воздействием напряжения, возникает защитная реакция. Иными словами, противостояние электрическому току начинает происходить в момент его непосредственного протекания через наше тело. В таких ситуациях происходит непросто сильное воздействие токов на организм человека, но и нарушение кровообращения, дыхания, сердечно-сосудистой и нервной системы и т.

п.

Электротравму предугадать нелегко, поскольку ее получение происходит не только при непосредственном контакте с токоведущими элементами, но и при взаимодействии с электрической дугой и шаговым напряжением.

Электротравматизм хоть и случается реже других видов производственных травм, но при этом находится на первых местах среди тех повреждений, которые оцениваются тяжелыми и приводящими к летальному исходу. Наибольший процент травм, вызванных влиянием электрического тока, происходит в процессе работы на электрических установках высокого напряжения (до 1000 В). Главной причиной электротравм служит частое использование именно таких типов электрических установок, а также недостаточная квалификация работников. Безусловно, существуют агрегаты с более высоким показателем напряжения (свыше 1000 В), но, как ни странно, в их эксплуатации поражения током редки. Такая закономерность объясняется высоким профессионализмом и компетентностью обслуживающего высоковольтные установки персонала.

Самыми распространенными причинами поражения током являются:

  • прямой телесный контакт с неизолированными токоведущими частями;
  • прикосновение к деталям электрического оборудования, изготовленным из металла;
  • прикосновение к неметаллическим элементам, находящимся под сильным напряжением;
  • взаимодействие с током шагового напряжения или с электрической дугой.

Классификация поражений электрическим током

Воздействие электрического тока при протекании через человеческий организм бывает термическим, электролитическим и биологическим.

    • Термическое воздействие – сильный нагрев тканей, что нередко сопровождается ожогами.
    • Электролитическое воздействие – разложение органических жидкостей, к которым относится и кровь.
    • Биологическое воздействие – нарушение биоэлектрических процессов, раздражение и возбуждение живых тканей, частое и беспорядочное сокращение мышц.

Поражения электротоком делятся на два основных вида:

  • Электротравмы – локальные поражения тканей или органов (ожоги, знаки, электрометаллизация).
    • Электрический ожог – итог сильного нагрева током (свыше одного ампера) тканей человека. Ожог, поражающий только кожный покров, называется поверхностным; повреждающий глубокие ткани тела является внутренним. Также электрические ожоги делятся по принципу возникновения: контактные, дуговые, смешанные.
    • Электрический знак внешне выглядит как серое или бледно-желтое пятно, напоминающее мозоль. Возникает данная травма в области контакта с токоведущим элементом. В основном, знаки не сопровождаются сильной болью и по прошествии небольшого количества времени сходят.
    • Электрометаллизация – явление, при котором кожа человека пропитывается металлическими микрочастицами. Это происходит в момент, когда металл под влиянием тока испаряется и разбрызгивается.
      Пораженная кожа приобретает цвет, соответствующий проникшим соединениям металла, и становится шероховатой. Процесс электрометаллизации не опасен, а эффект после него по истечении некоторого времени пропадает аналогично электрическим знакам. Куда более серьезные последствия имеет металлизация органов зрения.

Помимо ожогов, знаков и электрометаллизации в число электротравм также входит электроофтальмия и различные механические повреждения. Последние являются итогом непроизвольных сокращений мышц в момент протекания тока. К ним относятся сильные разрывы кожного покрова, кровеносных сосудов, нервов, а также вывихи и переломы. Электроофтальмия – явление, представляющее собой сильное воспаление глазных яблок после воздействия УФ-лучей электрической дуги.


  • Электрический удар выражается в форме сильного возбуждения живых тканей после воздействия на них электрического тока. Как правило, данное явление сопровождается беспорядочным судорожным сокращением мышц. Исход электроударов бывает разным, на основе чего они и делятся на пять видов:
    • без потери сознания;
    • с потерей сознания, сопровождающееся нарушением функционирования сердца и дыхания;
    • с потерей сознания, но без сбоев в работе сердечно-сосудистой системы и без нарушения дыхания;
    • клиническая смерть;
    • электрический шок.

Два последних вида стоит рассмотреть более подробно.

Клиническая смерть иначе называется также «мнимой» смертью, характеризующаяся длительностью в 6-8 минут. Данное явление считается переходным состоянием от жизни к смерти, которое сопровождается прекращением работы сердца и приостановлением дыхания. По прошествии вышеуказанного периода времени начинается необратимый процесс гибели клеток коры головного мозга, что заканчивается биологической смертью. 

Распознать мнимую смерть можно по следующим признакам:

    • фибрилляция сердца (т. е. разрозненное сокращение его мышечных волокон, сопровождающееся нарушением синхронной деятельности и насосной функции) или его полная остановка;
    • отсутствие пульса и дыхания;
    • синеватый цвет кожи;
    • расширенные зрачки без реагирования на свет, как следствие недостатка кислорода в коре головного мозга.

Электрический шок представляет собой тяжелую нервнорефлекторную реакцию человеческого организма на воздействие тока. Данное явление сопровождается сильными расстройствами дыхания, функционирования кровеносной и нервной системы и др.

Организм моментально реагирует на влияние электрического тока, вступая в фазу сильного возбуждения. В этот период происходит полная реакция на причинение боли, сопровождающаяся повышением артериального давления и другими процессами. Фаза возбуждения сменяется фазой торможения, которой свойственно истощение нервной системы, слабое дыхание, попеременное падение и учащение пульса, снижение артериального давления. Все перечисленные признаки приводят организм в состояние глубокой депрессии. Электрический шок может длиться как несколько десятков минут, так и несколько суток. Итог может быть полярно разным: либо полное выздоровление, либо необратимая биологическая смерть.


Предельные значения действия тока на человека

От показателя силы тока напрямую зависит его влияние на организм человека:

  • 0,6-1,5 мА при переменном токе (50Гц) и 5-7 мА при постоянном токе – ощутимый ток;
  • 10-15 мА при переменном токе (50Гц) и 50-80 мА при постоянном токе – не отпускающий ток, который в момент прохождения через организм провоцирует сильные судорожные сокращения мышц той руки, которая сжимает проводник;
  • 100 мА при переменном (50Гц) и 300 мА при постоянном токе – фибрилляционный ток, который приводит к фибрилляции сердца.

Влияние различных факторов на степень воздействия тока

Итог влияния электрического тока на организм человека также напрямую зависит от следующих факторов:

  • длительность протекания тока. То есть, чем дольше человек находился под воздействием, тем выше опасность и серьезней нанесенные травмы;
  • специфические особенности каждого организма в данный момент:
    масса тела, физическое развитие, состояние нервной системы, наличие каких-либо заболеваний, алкогольное или наркотическое опьянение и др.;
  • «фактор внимания», т.е. подготовленность к возможности получения электрического удара;
  • путь тока сквозь человеческое тело. Например, более серьезную опасность несет прохождение тока через сердце, легкие, мозг. В случае, если ток обошел жизненно важные органы, риск серьезных поражений резко снижается. На сегодняшний день зафиксирован самый популярный путь прохождения тока, который называется «петлей тока» — правая рука-ноги. Петли, отнимаемые работоспособность человека более чем на трое суток, представляют собой пути рука-рука (40%), правая рука-ноги (20%), левая рука-ноги (17%).

Знание влияния электрического тока на человеческий организм крайне необходимо. Это поможет Вам в чрезвычайных ситуациях оказать правильную медицинскую помощь пострадавшему.

Торговая сеть «Планета Электрика» обладает широким ассортиментом различных средств защиты при различных работах, с которым более подробно можно ознакомиться в нашем каталоге. 

«Объясните простым языком, что такое электричество, почему оно передается по проводам и больно бьется?» — Яндекс Кью

Популярное

Сообщества

Стать экспертом Кью

ЭлектричествоЭлектрикаТок

Ксан СтАркович

  ·

39,8 K

ОтветитьУточнить

Александр Стерлигов

Медицина

20,2 K

Врач-психиатр  · 19 дек 2017

Если объяснять простым языком, то электрический ток — это направленное движение заряженных частиц (электронов, ионов), которое создается изменением электрического поля (напряжением).  

Пришла в голову такая аналогия, которая немного проливает свет на суть электрического тока. Представьте большое сборище голодных солдат (электронов). И вдруг командир им начал кричать: «Там много вкусной и бесплатной еды!» (изменение электрического поля, создание напряжения). И для этого им построили специальную дорогу (электрический провод). После этого солдаты стройно побегут по этой дороге в поисках бесплатной еды, снося все преграды, нагревая поверхность дороги трением своих сапог, то есть будут совершать при этом работу. Эту дорогу построили так, что она ведет обратно в военную часть. И вот наши солдаты снова вернулись в часть, совершив работу на своём пути. Они готовы бежать снова и снова по этому порочному кругу, пока командиру будет хватать сил и энергии кричать «там много еды!» (то есть своим криком он создает напряжение, которое и приводит в движение солдат). Немного глупо, но представим просто таких солдат, чтобы понять электрический ток.

Так и электроны бегают по этому порочному кругу от источника тока по электрическим проводам обратно к источнику благодаря напряжению, которое создается разными путями (вращение генератора, солнечный свет падает на солнечную панель, химическая реакция в аккумуляторе и т. д.). Ну если быть точнее про напряжение: генератор/батарейка/солнечная панель под действием энергии (вращение турбины, химическая реакция, солнечный свет) перебрасывает электроны с одного своего полюса на другой. Тем самым создается избыток электронов на одном конце и недостаток на другом. Вот они и спешат установить равновесие, гуляя строем по проводам — сначала уходят от источника тока с одного его полюса, а потом возвращаясь к другому полюсу, выполнив нужную работу. Как только прекращается вращение генератора, иссякает химическая реакция в батарейке, то снижается и напряжение, уменьшается электрический ток, прекращается работа тока (горение лампочки, нагревание чайника и т.д).

А сейчас пришла в голову более простая аналогия: это ведь еще можно сравнить с обычным водяным насосом, который качает воду по замкнутому шлангу. Где вода — это электроны, движение воды — это электрический ток, шланг — провод, а насос — это источник тока.

Почему именно по проводам? Провод состоит из металлической жилы. Металлы состоят из атомов, которые находятся в виде металлических решеток, которые очень легко пропускают электроны (это как дороги построенные для солдат или как шланги из аналогии). Поэтому металлы проводят электрический ток.

19,4 K

Тимохин Санников

23 января 2019

Всегда была интересна аналогия с водой, и циркуляционным насосом по замкнутому кругу. Так же можно объяснить… Читать дальше

Комментировать ответ…Комментировать…

Asutpp

1,2 K

⚡Информационный сайт «ASUTPP». Статьи и рекомендации по ремонту электрооборудования…  · 6 февр 2020  · asutpp.ru

Отвечает

Юрий Макаров

Электричество представляет собой направленное движение заряженных частиц, к которым в любом материале относятся электроны или ионы, выступающие носителями заряда. Передаваться электрическая энергия может не только по проводам, но и по любым материалам, обладающим достаточной проводимостью для электротока. Передача электроэнергии происходит при возникновении на концах… Читать далее

Больше полезной информации по электрике вы можете найти на нашем сайте:

Перейти на asutpp.ru

Комментировать ответ…Комментировать…

Первый

Ahmeettovv Riishaatt

Гражданин России и житель города Санкт-Петербург мне 63 года Активный блогер и активный…  · 1 сент 2021

Электричество это заряженные частицы которые несут в себе источник энергии или работы говоря проще. Мы кушаем чтобы работать а электрон заряжается чтобы работать Мы ходим и ездием на работу по дороге или по шоссе Заряженные электроны они маленькие и их такая дорога не устраивает им нужен другой путь где их бы уважали и не толкали и пропускали на работу мы хоть идём… Читать далее

Комментировать ответ…Комментировать…

Виталий Микеров

5

Художник-Дизайнер. Биологию и Историю изучал сам. Религии и Философию тоже изучал сам…  · 19 июн 2021

Даже была такая шутка или история. Студен приходит на экзамен. И достает билет и там написано что такое электричество? Он говорит я учил учил. И экзаменатор говорит садись 5. Эх один человек знал что такое электричество, да и тот забыл. Была другая Шутка когда спросили электрика что такое электричество. Он ответил я не знаю что такое электричество но я знаю как… Читать далее

Комментировать ответ…Комментировать…

Александр

2

Люблю все делать своими руками   · 26 окт 2021

Представьте пластиковую трубу длиной 1 метр и диаметром 5 см, в которой бежит вода от точки а к точке б. Например протекает в ней 6 литров воды. Они могут протечь за 1 секунду, а могут за 20 секунд. Представили, а теперь : Диаметр трубы — это сечение провода. Чем больше сечение (диаметр), тем больше тока (воды) может пройти 6 литров воды — это количество тока в проводе. … Читать далее

Комментировать ответ…Комментировать…

leonidovantone

671

zen.yandex.ru/sciencecafe  · 22 янв 2019

Хочу предложить для прочтения свою статью о принципах работы токаklzzwxh:0000 В ней я постарался простым языком объяснить что такое напряжение, сила тока и сопротивление, а также немного коснуться чуть более сложных вещей. Если это было полезным- буду благодарен за подписку. Читать далее

Комментировать ответ…Комментировать…

Вы знаете ответ на этот вопрос?

Поделитесь своим опытом и знаниями

Войти и ответить на вопрос

1 ответ скрыто(Почему?)

Куда течёт электрический ток?

Современный человек отлично знаком с результатом работы тока в различных электроприборах, и редко задумывается о том, как, откуда и куда он течёт. Для тех, кто совсем немного знаком с электрикой и электроникой ответ будет прост и очевиден: от положительного полюса к отрицательному. Тем не менее, люди, которые знакомы с вопросом глубже, знают, что данное описание корректно не для всех ситуаций, что общепринятое понимание механизма несколько упрощено и на самом деле правильно ответить на подобный вопрос можно, только лишь уточнив его. Сегодня мы попытаемся рассказать читателям, как и почему возникла такая путаница.

Для начала следует вспомнить, что такое электроток. Справочники характеризуют его как направленное движение заряженных частиц. Сегодня принято считать, что в пределах цепи ток направлен от плюсового полюса источника питания к минусовому. Так работает любая техника на постоянном токе: радиоприёмники, фонарики, детские игрушки, пульты и даже те самые светодиодные светильники, которые через драйвер или трансформатор подключены к переменной сети. Вместе с тем, предполагается, что внутри самого источника питания – например, батарейки или аккумулятора – ток всё же идёт от минуса к плюсу. Почему так? Давайте разбираться.

 

 

 

Сегодня науке точно известно, что направление движения электронов во многом обусловлено материалом элементов цепи. Согласитесь, это звучит немного неожиданно, однако обо всём этом нам рассказывали в школе, просто другими словами. Так, если проводник изготовлен из металла, частицами, переносящими заряд, будут выступать электроны, несущие энергию от своего, отрицательного полюса к другому, положительному. И исходя из этого оказывается, что, вопреки сказанному ранее, электроны во внешней цепи движутся от минуса к плюсу. Доказать это довольно просто. Если взять любой диод, который по своей сути допускает прохождение тока только в одном направлении, и подключить так, как сегодня принято описывать направление течения электронов, он работать на будет. Полупроводники выполняют свою функцию только тогда, когда подключаются анодом к плюсовой клемме источника. Уже на основании одного этого можно понять, что в качестве направления электротока в цепи обычно принимают противоположное реальному движению электронов.

Путаница в понятиях сложилась лишь потому, что при открытии многих электрических явлений именно неверное описание казалось исследователям логичным. Задолго до изобретения лампочек учёные пытались работать с феноменом электричества. Широко известный американский общественный и научный деятель Бенджамин Франклин стал родоначальником так называемой унитарной теории электричества. Согласно его предположениям, это самое электричество является материей, а именно, жидкостью, лишённой веса, которая способна вытекать из одной точки и перетекать в другую, со временем накапливаясь в ней. Скорее всего, именно отсюда во многих языках мира и взялось слово «ток», связанное с глаголом «течь» – ведь текут обычно именно жидкости.

Франклин утверждал, что невесомая электрожидкость присутствует во всех телах, но выраженного заряда не имеет, а потому наэлектризоваться что-либо может только в том случае, когда наблюдается её недостаток или избыток. Логично, что нехватку учёный обозначил знаком минус, а излишек –знаком плюс. Сам того не понимая, он заложил этим тезисом основу понятий положительного и отрицательного зарядов. Для Франклина всё было просто и похоже на систему сообщающихся сосудов: когда в ней начинает наблюдаться дисбаланс, электрическая жидкость в нужном количестве перетекает от тела к телу, в обоих направлениях. В целом, хорошо понятную гипотезу о движении заряда опровергнуть было сложно, потому на многие годы представление осталось именно таким.

Примерно в то же время французский исследователь и известный физик своего времени Шарль Дюфе сделал пришёл к выводу, что в действительности существует целых две разновидности электричества, каждая из которых сама по себе вписывается в объяснения Франклина, но при контакте их эффект нейтрализуется. В доработанном виде эту теорию представил шотландский физик Роберт Симмер, который взял за основу опыты предшественника и дополнил их собственными объяснениями. Название теории полностью соответствовало сути – её нарекли дуалистической.

Для многих имя Симмера совершенно незнакомо, однако его можно считать «автором» самого знаменитого школьного эксперимента с эбонитовой палочкой. Хотя подобными играми баловались ещё древние греки, объяснение явлению смог дать только он. Известно, что учёный по жизни был склонен к переохлаждению и носил сразу две пары чулок: ближе к коже – тёплые, из шерсти, а поверх них, напоказ – шёлковые. И вот однажды он заинтересовался тем, почему они странно себя ведут после снятия. Когда Симмер снимал их вместе, а потом вытягивал один из другого, то видел, что и шёлк, и шерсть немного раздуваются, а затем слипаются друг с другом. При этом если взять пару чулок из одного материала, они будут отталкиваться. Его первые эксперименты были максимально просты: в одной руке находились шерстяные чулки, а в другой – шёлковые. При сближении рук одинаковые отталкивались, а разнородные моментально слипались. Сегодня мы знаем, что то же самое можно было бы сказать о полюсах магнитов, но тогда до идеи о связи электричества и магнетизма ещё никто не подозревал.

Зато благодаря работе Симмера стало понятно, что при натирании объекта с целью электризации заряженным становится не только это тело, но и то, которое его натирает. Дуалистическая теория поясняла, что в состоянии покоя в каждом теле в некотором количестве находятся сразу две невесомые электрические жидкости, противоположные по своему заряду. При этом в целом они нейтрализуют друг друга, но при изменении взаимных пропорций возникает электризация. Хотя гипотезы Франклина и Симмера не приводили учёный мир к единому мнению, обе они с необходимой для того времени достоверностью описывали видимое положение вещей, а потому сохранялись параллельно.

 

 

 

Следующий крупный этап в процессе выяснения правды наступил в 1799-том году. Задолго до появления на улицах электрических фонарных столбов, слово «столб» стало синонимом чего-то заряженного. Всё дело в том, что открытие явления электролиза с использованием вольтова столба более наглядно показало учёным, что заряды могут одновременно двигаться взаимно противоположно. Формально это было моментом торжества теории Симмера, но из-за нехватки информации об устройстве мира многие учёные не готовы были принять всё на веру. Многих смущало то, что при проведении эксперимента с электролизом на отрицательном электроде собиралось в два раза больше пузырьков водорода, чем на положительном – кислорода. Ввиду того, что формула Н2О ещё открыта не была, представлений о строении молекулы воды никто не имел, и это отчасти вносило трещину в дуалистическую теорию.

Спустя 21 год нашёлся учёный, который был гораздо решительнее предшественников. Его звали Андре-Мари Ампер, и он предложил Парижской академии наук устранить неоднозначность, приняв одно из направлений в качестве основного. В начале его работы над данным вопросом совершить выбор предполагалось просто на основании удобства, однако уже спустя несколько поставленных опытов Ампер сумел сформулировать единое правило, по которому можно было однозначно судить о направленности воздействия магнитов на электроток. Дабы избавиться от описания двух взаимно противоположных токов и избежать повторения, учёный решил однозначно, раз и навсегда, принять за основу направление движения положительного электричества. Именно этот момент считается формальной точкой отсчёта в отношении направленности электротока.

На основании тех же исследований британский физик Джеймс Клерк Максвелл сформулировал хорошо знакомое нам со школьной скамьи правило буравчика. Оно определяло направление магнитного поля катушки и вполне устраивало учёных, поскольку считалось адекватно описывающим реальность в тех координатах, которые ранее заложил Ампер. Вместе с тем, среди исследователей было немало и тех, кто даже при уважительном отношении к предшественникам продолжал критически смотреть на ситуацию. Англичанин Майкл Фарадей признавал, что пользоваться описанными правилами удобно, однако это не означает, что в природе всё так и есть. Уже после того, как он открыл явление электромагнитной индукции, возникла необходимость определить направление индуцированного тока, и на этом этапе сугубо теоретические и условные правила других исследователей не справлялись. Российский физик немецкого происхождения Эмилий Ленц сумел дать требуемую формулировку: если проводник из металла движется вблизи магнита или тока, внутри него возникает гальванический ток, направление которого таково, что, будь провод неподвижен, он бы пришёл в движение в сторону, противоположную исходному перемещению. Несмотря на длину разъяснения правила и его сложность для понимания при первом прочтении, именно оно утвердилось в качестве доминирующего.

И даже после открытия в 1897-ом году английским физиком Джозефом Джоном Томсоном электрона, указанная условность описания направления его движения сохранилась. Пусть природа задумала, что в проводнике или в вакууме должны перемещаться лишь электроны, человечество по-прежнему в качестве базового принимает противоположное направление – от плюса к минусу. Когда в начале ХХ-го века были изобретены электронные лампы, сразу же с оборудованием стали возникать определённые трудности. Тем не менее, даже это не заставило главные мировые умы пересмотреть подход. Ещё позже, с изобретением транзисторов путаница усилилась, но на первое место продолжало выноситься условное удобство. Сейчас люди уже привыкли считать, что там, где «плюс» энергии больше, чем там, где «минус», а потому она может переходить только в одном направлении, как во всё тех же сообщающихся сосудах у Франклина.

И хотя сегодня мы уже осведомлены о том, что данная условность не соответствует фактическому положению вещей, человечество успело изготовить такое количество электротехнической продукции, что внесение корректив в устоявшиеся принципы внесёт ещё большую сумятицу. Не пострадают разве что только те изделия, для которых полярность не имеет значения – это различные клеммники и наконечники, оснащение для переменного тока, а также различные провода и кабели. Всё остальное, в том числе, и бытовая техника, в которой много узлов преобразует энергию к 12 В или 5 В постоянного тока, может оказаться неработоспособной.

Напоследок хочется сказать о том, чему не уделено внимания выше: как же простому человеку понять, разобраться и запомнить, что и где находится, какой заряд куда течёт. Да, общепринятое направление движения электротока – это лишь некая условность, оправданная историей развития электротехники, и она противоположна реальному направлению перемещения электронов в металле, но в действительности всё это совершенно не принципиально. На самом деле, чтобы не прослыть невеждой следует руководствоваться простейшими принципами. Вернёмся к тому, что такое ток по определению – это направленное движение заряженных частиц. И вот тут самое главное: не спрашивайте себя, каких именно! Потому что правильный ответ – любых. Ими могут оказаться и негативно заряженные электроны, и положительные молекулы с атомами, и ионы вещества в растворе, и свободные электроны в полупроводниках, и даже так называемые «дырки». И всё это правильно, технически корректно. А потому вывод напрашивается довольно простой – ток течёт туда, где его «не хватает», то есть высказанный ранее принцип «от большего к меньшему» в действительности справедлив, безотносительно полярности перемещаемого по проводнику заряда. Остальные нюансы просто оказываются не важны.

Что такое электрический ток: основные понятия и характеристики

Содержание:

Электрический ток – это движение заряженных частиц в определенном направлении. Происходит подобное явление под влиянием поля. Частицами являются электроны, которые двигаются по проводнику и ионы, передвигающиеся в электролитной среде. Ионы бывают анионами и катионами. Проявляется ток в следующем:

  • нагрев проводника по которому он протекает, кроме сверхпроводников;
  • меняется химический состав, например, такое явление как электролиз;
  • появление магнитного поля. Ток считается направленным движением заряда с токопроводящей среде.

В статье будет рассказано все о таком  явлении, как ток. Подробнее будет рассказано об этом в двух видеороликах.

Электрический ток в проводах

Классификация

Если заряженные частицы движутся внутри макроскопических тел относительно той или иной среды, то такой ток называют электрический ток проводимости. Если движутся макроскопические заряженные тела (например, заряженные капли дождя), то этот ток называют конвекционный ток. Различают переменный (англ. alternating current, AC), постоянный (англ. direct current, DC) и пульсирующий электрические токи, а также их всевозможные комбинации. В таких понятиях часто слово «электрический» опускают. Постоянный ток — ток, направление и величина которого слабо меняются во времени.

Переменный ток — ток, величина и направление которого меняются во времени. В широком смысле под переменным током понимают любой ток, не являющийся постоянным. Среди переменных токов основным является ток, величина которого изменяется по синусоидальному закону.

Ток течет по проводам высоковольтных линий электропередач, ток вращает стартер и заряжает аккумулятор в нашем автомобиле, молния во время грозы — это тоже электрический ток.

Электрические разряды

В этом случае потенциал каждого конца проводника изменяется по отношению к потенциалу другого конца проводника попеременно с положительного на отрицательный и наоборот, проходя при этом через все промежуточные потенциалы (включая и нулевой потенциал).

В результате возникает ток, непрерывно изменяющий направление: при движении в одном направлении он возрастает, достигая максимума, именуемого амплитудным значением, затем спадает, на какой-то момент становится равным нулю, потом вновь возрастает, но уже в другом направлении и также достигает максимального значения, спадает, чтобы затем вновь пройти через ноль, после чего цикл всех изменений возобновляется.

Таблица электрический ток и его единицы измерения.

Квазистационарный ток

Это «относительно медленно изменяющийся переменный ток, для мгновенных значений которого с достаточной точностью выполняются законы постоянных токов» (БСЭ). Этими законами являются закон Ома, правила Кирхгофа и другие. Квазистационарный ток, так же как и постоянный ток, имеет одинаковую силу тока во всех сечениях неразветвлённой цепи. При расчёте цепей квазистационарного тока из-за возникающей э. д. с. индукции ёмкости и индуктивности учитываются как сосредоточенные параметры. Квазистационарными являются обычные промышленные токи, кроме токов в линиях дальних передач, в которых условие квазистационарности вдоль линии не выполняется.

[stextbox id=’warning’]Переменный ток высокой частоты — ток, в котором условие квазистационарности уже не выполняется, ток проходит по поверхности проводника, обтекая его со всех сторон. Этот эффект называется скин-эффектом.[/stextbox]

Пульсирующий ток

Ток, у которого изменяется только величина, а направление остаётся постоянным.

Вихревые токи (токи Фуко)

Замкнутые электрические токи в массивном проводнике, которые возникают при изменении пронизывающего его магнитного потока», поэтому вихревые токи являются индукционными токами. Чем быстрее изменяется магнитный поток, тем сильнее вихревые токи. Вихревые токи не текут по определённым путям в проводах, а замыкаясь в проводнике образуют вихреобразные контуры.

Вихревой ток

Существование вихревых токов приводит к скин-эффекту, то есть к тому, что переменный электрический ток и магнитный поток распространяются в основном в поверхностном слое проводника. Нагрев вихревыми токами проводников приводит к потерям энергии, особенно в сердечниках катушек переменного тока.

Для уменьшения потерь энергии на вихревые токи применяют деление магнитопроводов переменного тока на отдельные пластины, изолированные друг от друга и расположенные перпендикулярно направлению вихревых токов, что ограничивает возможные контуры их путей и сильно уменьшает величину этих токов.

П[stextbox id=’info’]ри очень высоких частотах вместо ферромагнетиков для магнитопроводов применяют магнитодиэлектрики, в которых из-за очень большого сопротивления вихревые токи практически не возникают.[/stextbox]

Характеристики

Исторически принято, что направление тока совпадает с направлением движения положительных зарядов в проводнике. При этом, если единственными носителями тока являются отрицательно заряженные частицы (например, электроны в металле), то направление тока противоположно направлению движения заряженных частиц. Скорость направленного движения частиц в проводниках зависит от материала проводника, массы и заряда частиц, окружающей температуры, приложенной разности потенциалов и составляет величину, намного меньшую скорости света.

Интересно почитать! Что такое варистор и где его применяют.

За 1 секунду электроны в проводнике перемещаются за счет упорядоченного движения меньше чем на 0,1 мм. Несмотря на это, скорость распространения собственно электрического тока равна скорости света (скорости распространения фронта электромагнитной волны). То есть то место, где электроны изменяют скорость своего движения после изменения напряжения, перемещается со скоростью распространения электромагнитных колебаний.

Разряд молнии – пример природного электричества

Основные типы проводников

В отличие от диэлектриков в проводниках имеются свободные носители нескомпенсированных зарядов, которые под действием силы, как правило разности электрических потенциалов, приходят в движение и создают электрический ток. Вольтамперная характеристика (зависимость силы тока от напряжения) является важнейшей характеристикой проводника. Для металлических проводников и электролитов она имеет простейший вид: сила тока прямо пропорциональна напряжению (закон Ома).

Таблица электрический ток в различных средах.
  • Металлы — здесь носителями тока являются электроны проводимости, которые принято рассматривать как электронный газ, отчётливо проявляющий квантовые свойства вырожденного газа.
  • Плазма — ионизированный газ. Электрический заряд переносится ионами (положительными и отрицательными) и свободными электронами, которые образуются под действием излучения (ультрафиолетового, рентгеновского и других) и (или) нагревания.
  • Электролиты — «жидкие или твёрдые вещества и системы, в которых присутствуют в сколько-нибудь заметной концентрации ионы, обусловливающие прохождение электрического тока». Ионы образуются в процессе электролитической диссоциации. При нагревании сопротивление электролитов падает из-за увеличения числа молекул, разложившихся на ионы. В результате прохождения тока через электролит ионы подходят к электродам и нейтрализуются, оседая на них. Законы электролиза Фарадея определяют массу вещества, выделившегося на электродах.

Существует также электрический ток электронов в вакууме, который используется в электронно-лучевых приборах.

Передача тока по проводам

Что такое ток, напряжение и сопротивление

Электрический ток ( I ) – это упорядоченное движение заряженных частиц. Первая мысль, которая приходит в голову из школьного курса физики – движение электронов. Безусловно. Однако электрический заряд могут переносить не только они, а, например, еще ионы, определяющие возникновение электрического тока в жидкостях и газах. Хочу предостеречь также от сравнения тока с протеканием воды по шлангу. (Хотя при рассмотрении Закона Кирхгофа такая аналогия будет уместна). Если каждая конкретная частица воды проделывает путь от начала до конца, то носитель электрического тока так не поступает.

Материал по теме: Что такое реле контроля.

Если уж нужна наглядность, то я бы привел пример переполненного автобуса, когда на остановке некто, втискиваясь в заднюю дверь, становится причиной выпадения из передней менее удачливого пассажира. Условиями возникновения и существования электрического тока являются:

  • Наличие свободных носителей заряда
  • Наличие электрического поля, создающего и поддерживающего ток.

Будем считать, что теперь про электрический ток Вы знаете все. Это, конечно, шутка. Тем более что еще ничего не сказано про электрическое поле, которое у многих ассоциируется с напряжением, что не верно. Электрическое поле – это вид материи, существующей вокруг электрически заряженных тел и оказывающее на них силовое воздействие. Опять же, обращаясь к знакомому со школы “одноименные заряды отталкиваются, а разноименные притягиваются” можно представить электрическое поле как нечто это воздействие передающее.

[stextbox id=’warning’]Это поле, равно как любое другое непосредственно ощутить нельзя, но существует его количественная характеристика – напряженность электрического поля.[/stextbox]

Существует множество формул, описывающих взаимосвязь электрического поля с другими электрическими величинами и параметрами. Я ограничусь одной, сведенной к примитиву: E=Δφ. Здесь:

  • E – напряженность электрического поля. Вообще это величина векторная, но я упростил все до скаляра.
  • Δφ=φ1-φ2 – разность потенциалов (рисунок 1).

Поскольку условием существования тока является наличие электрического поля, то его (поле) надо каким либо образом создать. Хорошо знакомые опыты электризации расчески, натирания тканью эбонитовой палочки, верчения ручки электростатической машины по вполне очевидным причинам на практике неприемлимы.

Электролиз в домашних условиях

Поэтому были изобретены устройства, способные обеспечивать разность потенциалов за счет сил неэлектростатического происхождения (одно из них – хорошо всем известная батарейка), получившие название источник электродвижущей силы (ЭДС), которая обозначается так: ε. Физический смысл ЭДС определяется работой, которую совершают сторонние силы, перемещая единичный заряд, но для того, чтобы получить первоначальное понятие что такое электрический ток, напряжение и сопротивление нам не нужно подробное рассмотрение этих процессов в интегральной и иных не менее сложных формах.

Напряжение ( U )

Наотрез отказываюсь продолжать заморачивать Вам голову сугубо теоретическими выкладками и даю определение напряжения как разности потенциалов на участке цепи: U=Δφ=φ1-φ2, а для замкнутой цепи будем считать напряжение равным ЭДС источника тока: U=ε. Это не совсем корректно, но на практике вполне достаточно. Сопротивление ( R ) – название говорит само за себя – физическая величина, характеризующая противодействие проводника электрическому току.  Формула, определяющая зависимость напряжения, тока и сопротивления называется закон Ома. Этот закон рассматривается на отдельной странице этого раздела.

Кроме того, сопротивление зависит от ряда факторов, например, материала проводника. Данные эти справочные, приводятся в виде значения удельного сопротивления ρ, определяемого как сопротивление 1 метра проводника/сечение. Чем меньше удельное сопротивление, тем меньше потери тока в проводнике.

Источники электрической энергии

Соответственно сопротивление проводника длиной L и площадью сечения S, будет составлять R=ρ*L/S. Непосредственно из приведенной формулы видно, что сопротивление проводника также зависит от его длины и сечения. Температура тоже оказывает влияние на сопротивление. Несколько слов про единицы измерения тока, напряжения, сопротивления. Основные единицы измерения этих величин следующие:

  • Ток – Ампер (А)
  • Напряжение – Вольт (В)
  • Сопротивление – Ом (Ом).

Это единицы измерения интернациональной системы (СИ) не всегда удобны. На практике применяются из производные (милиампер, килоом и пр.). При расчетах следует учитывать размерность всех величин, содержащихся в формуле. Так, если Вы, в законе Ома умножите ампер на килоом, то напряжение получите совсем не вольтах.

Интересно по теме: Как проверить стабилитрон.

Терминология

Когда мы произносим словосочетание «электрический ток», то обычно имеем ввиду самые разные проявления электричества. Ток течет по проводам высоковольтных линий электропередач, ток вращает стартер и заряжает аккумулятор в нашем автомобиле, молния во время грозы — это тоже электрический ток. Электролиз, электросварка, искры статического электричества на расческе, по спирали лампы накаливания течет ток, и даже в крохотном карманном фонарике через светодиод течет крохотный ток. Что и говорить о нашем сердце, которое также генерирует небольшой электрический ток, особенно это заметно во время прохождения процедуры ЭКГ.

Переменное магнитное поле

В физике электрическим током принято называть упорядоченное движение заряженных частиц и в принципе любых носителей электрического заряда. Движущийся вокруг атомного ядра электрон — это тоже ток. И заряженная эбонитовая палочка, если держать ее в руке и двигать из стороны в сторону — также станет источником тока: не равный нулю заряд есть и он движется.

Физические аналогии между течением воды в системе водоснабжения и электрическим током: Электропроводка и трубопровод. Ток течет по проводам бытовых электроприборов питающихся от розетки — электроны перемещаются туда-сюда 50 раз за секунду — это называется переменным током. Высокочастотные сигналы внутри электронных приборов — это тоже электрический ток, поскольку электроны и дырки (носители положительного заряда) перемещаются внутри схемы. Любой электрический ток порождает своим существованием магнитное поле. Вокруг проводника с током оно обязательно присутствует. Не существует магнитного поля без тока и тока без магнитного поля.

Даже если магнитного поля вокруг тока не наблюдается, это лишь значит что магнитные поля двух токов в момент наблюдения взаимно скомпенсированы, как в двужильном проводе любого электрического чайника — переменные токи в каждый момент направлены в противоположные стороны и текут параллельно друг другу — их магнитные поля друг друга нейтрализуют. Это называется принципом наложения (суперпозиции) магнитных полей.

Практически для существования электрического тока необходимо наличие электрического поля, потенциального или вихревого. Исключительно редко заряды перемещаются чисто механическим образом (как например в генераторе Ван Де Граафа — наэлектризованной резиновой лентой). В электрическом поле заряженная частица испытывает действие электрической силы, которая у источников тока называется ЭДС — электродвижущая сила. ЭДС измеряется в вольтах как и напряжение между двумя точками электрической цепи. Чем больше напряжение приложенное к потребителю — тем больший электрический ток это напряжение способно вызвать.

Магнитное поле от электрического разряда

Переменное напряжение порождает в проводнике, к которому оно приложено, переменный ток, поскольку электрическое поле, приложенное к носителям заряда, будет в этом случае также переменным. Постоянное напряжение — условие существования в проводнике тока постоянного. Высокочастотное напряжение (изменяющее свое направление сотни тысяч раз за секунду) также способствует переменному току в проводниках, но чем выше частота — тем меньше носителей заряда участвуют в создании тока в толще проводника, поскольку электрическое поле действующее на заряженные частицы вытесняется ближе к поверхности, и получается что ток течет не в проводнике, а по его поверхности. Это называется скин-эффект.

Электрический ток может существовать в вакууме, в проводниках, в электролитах, в полупроводниках и даже в диэлектриках (ток смещения). Правда в диэлектриках постоянного тока быть не может, поскольку в них заряды не имеют возможности к свободному перемещению, а способны лишь смещаться в пределах внутримолекулярного расстояния от своего первоначального положения под действием приложенного электрического поля.

[stextbox id=’warning’]Настоящий электрический ток всегда предполагает возможность свободного перемещения электрических зарядов под действием электрического поля. Смотрите – условия существования электрического тока. В металлических проводниках электрический ток представляет собой движение «свободных» электронов, причем электроны движутся в направлении, противоположном условному направлению тока (т. к. за направление тока условно принято направления движения зарядов).[/stextbox]

Электрический ток  в газах представляет собой движение положительных ионов в одном направлении, а электронов (и отрицательных ионов) в другом направлении. Наконец, электрический ток в электролитах представляет собой движение существующих в жидкости положительных и отрицательных ионов в противоположных направлениях. Сила электрического тока — количество электричества, прошедшее через все поперечное сечение тока за 1 сек., зависит, с одной стороны, от количества движущихся зарядов, а с другой — от средней скорости их регулярного движения. В металлических проводниках количество движущихся зарядов (свободных электронов) чрезвычайно велико (порядка 1023 в 1 см3), но зато средняя скорость регулярного движения очень мала (при самых сильных токах, которые может выдержать проводник, эта средняя скорость имеет величину порядка сантиметра в секунду). Обычно несколько меньше количество движущихся зарядов в жидкостях и соответственно их средние скорости несколько больше.

В газах же вследствие их гораздо меньшей плотности и вследствие того, что только небольшая доля всех молекул газа оказывается ионизированной, количество движущихся зарядов гораздо меньше, но зато средние скорости движения электронов и ионов гораздо больше, чем в металлических проводниках, и достигают сотен и даже тысяч километров в секунду. Понятие “электрический ток” ввел итальянский физик Алессандро Вольта. Электрический ток, или по его версии “электрический флюид” протекал в замкнутой цепи, соединяющей металлическим проводником крайние кружки вольтова столба.

“Вотльтов столб” (1800 г.) был первый источник электричества неэлектростатического типа (источник постоянного электрического тока), который состоял из чередующихся между собой медных и цинковых кружков, разделенных суконными прокладками, смоченными подкисленной водой или кислотой. Существование неизменного высокого потенциала на вольтовом столбе было явлением для того времени совершенно новым. Это был первый химический источник электричества, потенциал которого был постоянен во времени и не требовал каких-либо приемов электризации для его возобновления.

Вольтов столб, составленный из большого количества кружков, имел на концах достаточно высокий потенциал, который можно было обнаружить не только измерительными приборами (в частности электроскопом), но и прикоснувшись к крайним кружкам руками. При этом ощущался сильный электрический удар, как от лейденской банки.  Открытие Вольты очень быстро распространилось в физике, стало предметом дальнейших исследований. В 1800 г. ученые-физики с помощью вольтова столба обнаружили электрохимическое действие тока, и в частности разложение под действием тока воды на кислород и водород. Опыты с гальваническими элементами позволили обнаружить, кроме химических, и другие новые свойства тока, в том числе его тепловое и магнитное действие.

Важное по теме. Как проверить конденсатор.

Французский физик А. М. Ампер посвятил ряд своих работ изучению связи электрического тока и магнетизма. Он обнаружил, что два проводника с током испытывают взаимное воздействие — притяжение или отталкивание в зависимости от направления в них токов. Своими работами он заложил основы электродинамики. Он предложил термин “электрический ток” и ввел понятие о его направлении, совпадающем с движением положительного электричества. В честь А. М. Ампера названа единица измерения электрического тока.  Ампер является одной из семи основных единиц системы СИ.

Электрический ток обладает рядом свойств, которые могут быть эффективно использованы во многих практических случаях. К таким свойствам относятся трансформация простыми техническими средствами энергии электрического тока в энергию других видов (тепловую, световую, механическую, химическую) и возможность передачи ее на большие расстояния, быстрота распространения.

Заключение

Рейтинг автора

Написано статей

Более подробно о том, что такое ток, рассказано в статье Что такое электрический ток. Если у вас остались вопросы, можно задать их в комментариях на сайте. Если у вас остались вопросы, можно задать их в комментариях на сайте. А также в нашей группе ВК можно задавать вопросы и получать на них подробные ответы от профессионалов. Для этого приглашаем читателей подписаться и вступить в группу.

В завершение статьи хотелось бы выразить благодарность источникам информации для подготовки материала:

www.electricalschool.info

www.electrik.info

www.elektal.com.ua

www.allatra-science.org

www.eltechbook.ru

www.meanders.ru

Предыдущая

ТеорияЗаконы Кирхгофа простыми словами: определение для электрической цепи

Следующая

ТеорияКак работает выпрямитель напряжения

Проектируем электрику вместе: Что такое электрический ток?

Что такое электричество?.. Определение электричества.. Атомный уровень материи.. Атом — строительный «кирпичик» вещества.. Строительные элементы атома — протоны, нейтроны и электроны.. Свободные электроны.. Ионы.. Электрическое поле.. Электрический ток..

Мы живем в электрическом веке. Невозможно представить нашу жизнь без электричества — оно, буквально, окружает нас: это освещение и тепло в наших домах, наши сотовые телефоны и компьютеры, микроволновки и кондиционеры. Это трамваи и троллейбусы, поезда в метро и электрички. Это навигационные приборы на корабле и в подводной лодке, самолете и космической ракете… Перечислять можно сколь угодно долго…

Даже в природе мы сталкиваемся со всевозможными проявлениями электричества, от молнии в грозу до нервных импульсов в нейронах нашего организма.
 
Но что такое есть электричество?

Это очень непростой вопрос. Поскольку – если не копать глубже – невозможно получить простой и окончательный ответ, кроме общих абстрактных представлений о том, как электричество проявляется в окружающем мире.
Есть много определений электричества. Вот одно из них:

Электричество есть свойство материи (вещества), обусловленное взаимодействием и движением электрических зарядов.

Определение, которое не лучше и не хуже многих других, зато короткое. Надо сказать, что любые определения электричества будут поверхностными, поскольку не раскрывают базовых понятий. Что такое электрические заряды? Какими свойствами они обладают? Как электрические заряды взаимодействуют? Почему они движутся? Как обнаружить это движение?…

Строительные «кирпичики» вещества

Чтобы получить ответы на эти и многие другие вопросы, мы должны изменить масштаб, перейти с внешнего видимого уровня на внутренний — атомный уровень.

Атом – один из основных строительных «кирпичиков» материи, как и жизни тоже. Атомы существуют в более чем ста различных формах, как химические элементы: водород, углерод, кислород, медь и т. д. Атомы многих видов могут объединяться и создавать молекулы, из  которых строятся различные вещества, которые мы можем физически увидеть и потрогать.

Атомы представляют собой крошечные частицы, размером не более 300 пм (это 300 ·10 -12 или 3 ·10 -10 или 0,0000000003 м). Однако даже атом не достаточно мал, чтобы объяснить, как получается электричество.   Мы должны нырнуть глубже, на следующий уровень и посмотреть, из каких строительных элементов, в свою очередь, состоит сам атом.

Строительные элементы атома

Атом представляет собой комбинацию из трех различных частиц: электронов, протонов и нейтронов. Каждый атом имеет центральное ядро, в котором протоны и нейтроны плотно упакованы вместе. Ядро окружает группа орбитальных электронов (рис. 1).
До 30-х годов прошлого века думали, что электроны вращаются на отдельных орбитах вокруг ядра атома, как планеты солнечной системы вокруг Солнца. Дальнейшие исследования показали, что орбиты – не совсем подходящее понятие для описания электронов. Сегодня считается, что электроны существуют, как бы в «облаке», что окружает атомное ядро.
В любой заданный момент электрон имеет некоторую статистическую вероятность нахождения его где-то в «облаке». Это пока все, что можно об этом сказать.

Каждый атом должен иметь, по крайней мере, один протон. Число протонов в атоме определяет, какой химический элемент представляет атом. Например, атом с одним протоном – это атом водорода. Атом с 29 протонами – медь. Атом с 94 протонами – плутоний…
Таким образом, количество протонов в атоме определяет его атомный номер в таблице Менделеева.
Партнеры протона по ядру – нейтроны играют важную роль в стабильности ядра и определяют изотопы атомов. Они не имеют решающего значения для понимания электричества.

Электроны – именно они имеют решающее значение для объяснения электрического тока. Поскольку электроны и протоны – противоположно заряженные частицы (электроны несут отрицательный заряд, а протоны – положительный), то в стабильном состоянии каждый атом предпочитает иметь одинаковое число электронов и протонов. Нейтроны, оправдывая свое название, являются нейтральными, они не имеют заряда. Следовательно, атом в целом электрически нейтрален. В стабильном, уравновешенном состоянии, у атома всегда будет такое же число электронов, как и протонов. Так, ядро с 29 протонами (атом меди) окружено «облаком» из 29 электронов.

Свободные электроны

Вкратце разобравшись со строением атома, мы вплотную подошли к объяснению природы электрического тока в проводниках.

Не все электроны вечно связаны с атомом. Электрон, расположенный на внешней оболочке атома (по причине его слабой связи с ядром) под воздействием внешней силы может покинуть орбиту атома и стать свободным (рис. 2). Появляется атом с отсутствующим электроном, который называется ионом. Из-за отсутствия электрона на внешней оболочке указанный ион становится положительно заряженным.

Этот ион может привлечь на свою внешнюю оболочку другие электроны, которые ранее были отделены от любого другого атома и, следовательно, этот ион снова становится нейтральным атомом. Потом он снова может стать положительно заряженным…

Электроны, которые движутся в проводнике от атома к атому случайным образом, называются свободными электронами. Наличие свободных электронов — вот ключ к пониманию природы электрического тока в проводнике.

Если теперь к проводнику приложить электрическое поле (рис. 3), то свободные электроны начинают дрейфовать в определенном направлении в соответствии с полярностью приложенного напряжения.


Такое направленное (упорядоченное) движение электронов в проводнике под действием электрического поля называется электрическим током.

Какое направление движения тока принимается для расчетов и каково действительное направление движения электронов? Что такое постоянный и переменный ток?… Об этом и многом другом — в следующих уроках. 

Похожие статьи: 1. Взаимодействие электрических зарядов. Закон Кулона
                              2. Направление электрического тока
                              3. Постоянный и переменный ток
                              4. Проводники и изоляторы. Полупроводники
                              5. О скорости распространения электрического тока
                              6. Электрический ток в жидкостях 
                              7. Проводимость в газах
                              7. Электрический ток в вакууме
                              8. О проводимости полупроводников
 

напряжение, сила тока и сопротивление, постоянный и переменный ток

На сегодняшней встрече мы поведем разговор об электричестве, которое стало неотъемлемой частью современной цивилизации. Электроэнергетика вторглась во все сферы нашей жизни. А присутствие в каждом доме бытовых приборов, использующих электрический ток настолько естественная и неотъемлемая часть быта, что мы принимаем это как должное.

Итак, вниманию наших читателей предлагаются основные сведения об электрическом токе.

Что такое электрический ток

Под электрическим током понимают направленное движение заряженных частиц. Вещества, содержащие достаточное количество свободных зарядов, называют проводниками. А совокупность всех устройств, соединенных между собой помощью проводов называют электрической цепью.

В повседневной жизни мы используем электричество, проходящее по металлическим проводникам. Носителями заряда в них являются свободные электроны.

Обычно они хаотично мечутся между атомами, но электрическое поле вынуждает их двигаться в определенном направлении.

Как это происходит

Поток электронов в цепи можно сравнить с потоком воды, ниспадающей с высокого уровня на низкий. Роль уровня в электрических цепях играет потенциал.

Для Протекания тока в цепи на её концах должна поддерживаться постоянная разность потенциалов, т.е. напряжение.

Его принято обозначать буквой U и измерять в вольтах (B).

Благодаря приложенному напряжению в цепи устанавливается электрическое поле, которое и придаёт электронам направленное движение. Чем больше напряжение, тем сильнее электрическое поле, а значит и интенсивность потока направленно движущихся электронов.

Скорость распространения электрического тока равна скорости установления в цепи электрического поля, т. е. 300 000 км/с, однако скорость электронов едва достигает лишь нескольких мм в секунду.

Принято считать, что ток течёт от точки с большим потенциалом, т. е. от (+) к точке с меньшим потенциалом, т. е. к (−). Напряжение в цепи поддерживается источником тока, например батарейкой. Знак (+) на её конце означает, недостаток электронов, знак (−) их избыток, поскольку электроны — носители именно отрицательного заряда. Как только цепь с источником тока становиться замкнутой, электроны устремляются от места, где их избыток, к положительному полюсу источника тока. Их путь пролегает через провода, потребители, измерительные приборы и другие элементы цепи.

Обратите внимание, направление тока противоположно направлению движения электронов.

Просто направление тока по договоренности учёных определили до того как была установлена природа тока в металлах.

Некоторые величины, характеризующие электрический ток

Сила тока. Электрический заряд, проходящий через поперечное сечение проводника за 1 сек, называют силой тока. Для её обозначения используют букву I, измеряют в амперах (A).

Сопротивление. Следующая величина, о которой необходимо знать — это сопротивление. Оно возникает из-за столкновений направленно движущихся электронов с ионами кристаллической решетки. В результате таких столкновений электроны передают ионам часть своей кинетической энергии. В результате чего проводник нагревается, а сила тока уменьшается. Сопротивление обозначается буквой R и измеряется в омах (Ом).

Сопротивление металлического проводника тем больше, чем длиннее проводник и меньше площадь его поперечного сечения. При одинаковой длине и диаметре провода наименьшим сопротивлением обладают проводники из серебра, меди, золота и алюминия. По вполне понятным причинам на практике используют провода из алюминия и меди.

Мощность. Выполняя расчёты для электрических цепей, иногда требуется определить потребляемую мощность (P).

Для этого следует силу тока, протекающую по цепи умножить на напряжение.

P=IU

Единицей измерения мощности служит ватт (Вт).

Постоянный и переменный ток

Ток, даваемый разнообразными батарейками и аккумуляторами, является постоянным. Это означает, что силу тока в такой цепи можно изменять лишь по величине, меняя различными способами её сопротивление, а его направление при этом сохраняется неизменным.

Но большинство электробытовых приборов потребляют переменный ток, т. е. ток величина и направление которого непрерывно изменяются по определенному закону.

Он вырабатывается на электростанциях, а затем через линии высоковольтных передач попадает в наши дома и на предприятия.

В большинстве стран частота изменения направления тока равна 50 Гц, т. е происходит 50 раз в секунду. При этом каждый раз сила тока постепенно нарастает, достигает максимума, затем убывает до 0. Затем этот процесс повторяется, но уже при противоположном направлении тока.

В США все приборы работают на частоте 60 Гц. Интересная ситуация сложилась в Японии. Там на одной трети страны используют переменный ток с частотой в 60 Гц, а на остальной части — 50 Гц.

Осторожно — электричество

Поражения электрическим током можно получить при использовании электробытовых приборов и от ударов молнии, поскольку человеческий организм хороший проводник тока. Нередко электротравмы получают, наступив на лежащий на земле провод или отодвинув руками отвисшие электрические провода.

Напряжение свыше 36 В считается опасным для человека. Если через тело человека пройдет ток всего лишь в 0,05 А, он может вызвать непроизвольное сокращение мышц, которое не позволит человеку самостоятельно оторваться от источника поражения. Ток в 0,1 А смертелен.

Ещё опаснее переменный ток, поскольку оказывает более сильное воздействие на человека. Этот наш друг и помощник в ряде случаев превращается в беспощадного врага, вызывая нарушение дыхания и работу сердца, вплоть до его полной остановки. Он оставляет страшные метки на теле в виде сильнейших ожогов.

Как помочь пострадавшему? Прежде всего, отключить источник поражения. А затем уже позаботиться об оказании первой медицинской помощи.

Наше знакомство с электричеством подходит к концу. Добавим лишь несколько слов о морских обитателях, обладающих «электрическим оружием». Это некоторые виды рыб, морской угорь и скат. Самым опасным из них является морской угорь.

Не стоит подплывать к нему на расстояние менее 3 метров. Удар его не смертелен, но сознание можно потерять.

Автор: Драчёва Светлана Семёновна


Если это сообщение тебе пригодилось, буда рада видеть тебя в группе ВКонтакте. А ещё — спасибо, если ты нажмёшь на одну из кнопочек «лайков»:


Электрический ток Определение и значение

  • Основные определения
  • Викторина
  • Примеры
  • Британский

Показывает уровень сложности слова.

Сохрани это слово!

Показывает уровень сложности слова.


сущ. Электричество.

скорость потока электрического заряда во времени в направлении, в котором движется положительный заряд, и имеющая величину, равную количеству заряда в единицу времени: измеряется в амперах.

ВИКТОРИНА

Сыграем ли мы в «ДОЛЖЕН» ПРОТИВ. «ДОЛЖЕН» ВЫЗОВ?

Должны ли вы пройти этот тест на «должен» или «должен»? Это должно оказаться быстрым вызовом!

Вопрос 1 из 6

Какая форма используется для указания обязательства или обязанности кого-либо?

Также называется током, электричеством.

Происхождение электрического тока

Впервые записано в 1830–1840 гг. лампа, электрическая смещение, электрический угорь

Dictionary.com Полный текст На основе Random House Unabridged Dictionary, © Random House, Inc. 2022

Как использовать электрический ток в предложении

  • Этот поток заряженных частиц производит электрический ток, который вытекает из батареи для питания всего, что нуждается в питании. .

    Новые литий-металлические аккумуляторы приведут к переходу на электромобили|Кэти Маклин|24 февраля 2021 г.|MIT Technology Review

  • Самые важные весенние тренировочные вопросы национальных сборных все на поляхОщущение общего опыта, даже хотя и обрабатывается совершенно по-разному, через каждого, кто помнит, проходит, как слабый электрический ток.

    Ничто не может испортить первый день весенних тренировок. (Хотя covid, безусловно, дает ему шанс.)|Thomas M. Boswell|18 февраля 2021 г.|Washington Post

  • Новое замедленное видео предлагает лучший вид на доли секунды столкновения электрических токов, которые создают вспышка молнии.

    Рождение молнии было заснято на видео|Мария Темминг|8 февраля 2021|Новости науки

  • По словам Нойберта, это может вызвать очень короткий, но мощный всплеск электрического тока.

    Датчики космической станции увидели, как формируется странная «голубая струя» молнии|Мария Темминг|2 февраля 2021|Новости науки для студентов

  • Эти реакции создают заряды, которые позволяют батарее обеспечивать электрический ток.

    Батареи не должны воспламеняться|Кэролин Уилке|16 апреля 2020 г.|Новости науки для студентов

  • Нынешние и бывшие сотрудники разведки заявляют, что Северная Корея уже давно является приоритетной целью для американских шпионов.

    Sony подверглась второму взлому?|Шейн Харрис|8 января 2015|DAILY BEAST

  • Один из ее высокопоставленных чиновников является нынешним министром внутренних дел в Багхаде.

    Что иранские похороны говорят нам о войнах в Ираке|IranWire|6 января 2015|DAILY BEAST

  • Примерно через неделю она бросила JSwipe и нашла своего нынешнего бойфренда, нееврея, на OkCupid.

    Моя неделя в еврейском Tinder|Эмили Шайр|5 января 2015 г.|DAILY BEAST

  • Судя по текущим цифрам, на этот вариант тоже будет большой спрос.

    У мужчин когда-нибудь будут дети без женщин|Саманта Аллен|3 января 2015|DAILY BEAST

  • Джетт отмечает, что при нынешнем президенте и его предшественнике пост посла Белиза достался соседям по комнате в колледже.

    Посольства США всегда выставлялись на продажу|Уильям О’Коннор|2 января 2015|DAILY BEAST

  • Но он омрачил все это настолько неуправляемым характером, что в Париже вошло в обиход пословицу: «Взрывной, как Гарнаш». »

    Лето Св. Мартина|Рафаэль Сабатини

  • Это — а также отсутствие существующих учреждений и текущих проблем — является главной заботой нынешнего века.

    Спасение цивилизации|H. Г. (Герберт Джордж) Уэллс

  • В текущем году было обнаружено, что невозможно читать даже газету!

    Эдинбургский журнал Blackwood, № CCCXXXIX. Январь 1844 г. Том. LV.|Various

  • Словно током, известные аккорды Трагалы будили его слушателей — все столпились вокруг певца.

    Эдинбургский журнал Блэквуда, том 60, № 372, октябрь 1846 г.|Разное

  • Его игра на барабанах перед Кафе де ла Лож произвела ошеломляющее впечатление.

    Веселые приключения Аристида Пужоля|Уильяма Дж. Локка Полное цифровое издание 2012 г. © William Collins Sons & Co. Ltd. 1979, 1986 © HarperCollins Издательства 1998, 2000, 2003, 2005, 2006, 2007, 2009, 2012

    Электричество — Электрический ток — Физика 299

    Электричество — Электрический ток — Физика 299

    «Когда я оказываюсь в компании ученых, я чувствую себя жалким священником, заблудшим ошибиться в гостиной, полной герцогов»
    У. Х. Оден

    Электрический ток
    • Электрический ток равен скорости прохождения заряда неподвижная точка в пространстве.


    • Ток измеряется в Амперах:
      1 Ампер = 1 Кулон/секунда

      Хотя из вышеприведенного определения видно, что Ампер определяется в терминах кулона, на самом деле это ампер, который — основная единица, кулон — производная единица. Ампер это определяется через силу, действующую между двумя параллельными проводами, несущими текущий, как мы увидим позже.
    • Важно понимать, что значение тока постоянной, независимо от сечения проводника. Если если бы это было не так, то заряд «скапливался бы» в точках вдоль проводник.

    • Когда вы щелкаете выключателем, лампочка мгновенно загорается. На самом деле течение движется со скоростью, близкой к скорости легкий. Однако носители заряда, электроны в металлическая проволока движется с гораздо меньшей скоростью — скоростью дрейфа.
      Рассмотрим провод длиной l, сечением A, проводимостью n электронов в единице объема. Ток в проводе может быть написано,

    где e — заряд электрона и v d — дрейфовая скорость.

    • Плотность тока, Дж (А/м 2 ) определяется как

    физически, J представляет движение заряда в определенном месте внутри проводник, напр. когда А большое J маленькое, когда А маленькое J большой.
    Общее соотношение между I и J:


    Ток – это поток J через поверхность.

    Важно: Текущий, Я, является скалярная величина, тогда как J является вектором. у меня есть «смысл» в том, что мы рисуем стрелки, чтобы представить его «направление», но не подчиняется правилам вектора алгебра.


    • Историческая причуда. Направление тока определяется как направление в который будет двигаться положительный заряд. Но в твердом металлическом проводники носители заряда электроны (отрицательные заряды) которые на самом деле движутся в противоположном направлении. Отрицательный заряды, движущиеся справа налево, в точности эквивалентны положительным заряды движутся слева направо.

    Сопротивление

      • В металлических проводниках электрическое поле и плотность тока направлены в одну сторону и пропорциональны друг друга,

      , где ρ — удельное сопротивление проводника — характеристика проводник. Электропроводность проводящего материала равна определяется формулой σ = 1/ρ.
      Для однородного проводника длиной l, поперечного сечения A имеем E = V/l и J = i/A, так что

      Сопротивление проводника R, определяется как,


      Сопротивление измеряется в омах (Ом), тогда удельное сопротивление единицы ом.метр и проводимость (ом.метр) -1

      • Важно: Соотношение V = ИК НЕ Закон Ома !

      Закон Ома:

      «Если соотношение напряжение на проводнике к току через она постоянна для всех напряжений, то проводник подчиняется закону Ома»


      Закон Ома выполняется для металлических проводников, но не для таких устройств, как транзисторы, диоды и т. д. Соотношение V = IR всегда можно использовать для определить сопротивление при каком-то конкретном I и V для любого устройства.


      • Даже в проводниках ток будет течь только между двумя точками А и Б, когда

      1. Существует разность потенциалов между A и B (производящая электрическое поле, которое заставляет заряды двигаться) и,
      2. A и B являются частью полной цепи.


      Мощность

      • Предположим, заряд dq движется из точки A в точку B, где разность потенциалов между A и B равна V AB , то энергия, выделяемая за время dt, равна
      • .

      , так что скорость, с которой энергия переданная (мощность), P, дается,

      В единицах измерения мы можем сказать, что Ампер x Вольт = Ватт.

      • Форма «высвобождаемой» энергии зависит от электрический компонент, расположенный между A и B, для например,
        • Двигатель — высвобожденная механическая энергия (работа)
        • Аккумулятор — химическая энергия, хранящаяся в аккумуляторе
        • Сопротивление — выделяется тепловая энергия (тепло)

        Электродвижущая сила — «ЭДС»

        • При обсуждении электрических цепей вам может встретиться термин «ЭДС» — электродвижущая сила. Важно понимать, что «ЭДС» НЕ сила!
        • Если устройство имеет «ЭДС», оно может поддерживать разность потенциалов (напряжение). Так, например, батарея поддерживает ЭДС между положительной и отрицательной терминалы.
        • ЭДС устройства можно определить как ε = dW/dq, где dW работа, совершенная положительным зарядом dq при его взятии через разность потенциалов устройства. в случай простой схемы с батареей (см. выше) в качестве заряд проходит по внешней (по отношению к аккумулятору) цепи ее теряет энергию. В цепи выше появляется энергия как тепло и свет в лампочке. Когда заряд возвращает аккумулятору ЭДС аккумулятора восполняет свою энергию.
        • На этом вводном уровне мы можем рассмотреть ЭДС «источник» (аккумулятор, генератор и т. д.) должен быть точно эквивалентен к напряжению, обеспечиваемому источником.
        • Направление ЭДС всегда представляет направление a положительный заряд будет перемещаться во внешней цепи. См. схему справа. Направление ЭДС – это важным фактором, когда мы используем законы Кирхгофа для анализа схемы.


          Внутреннее сопротивление

          • Все ЭДС — батареи, генераторы и т.д. — и электрические измерительные приборы — амперметры, вольтметры и т.п. — имеют «внутреннее сопротивление».
          • Что касается анализа цепи обеспокоены тем, что эти внутренние сопротивления могут быть просто рассматривать как резисторы, включенные последовательно с «идеальным» ЭДС/метр.
          • Для амперметров (токоизмерительных приборов) целью является иметь как можно более низкое внутреннее сопротивление, чтобы ток не влияет.

          • Для вольтметра внутреннее сопротивление должно быть как можно больше насколько это возможно.

          В: Имеет ли свет массу?
          А: Конечно нет. Это даже не католик!!!


          Dr. C.L. Davis
          Физический факультет
          Университет Луисвилля
          электронная почта : [email protected]
           

          Электрический ток: определение, формула и единицы

          Электрический ток: определение, формула и единицы | StudySmarter

          Выберите язык

          Предлагаемые языки:

          Европа

          английский (DE) английский (Великобритания)

          StudySmarter — универсальное учебное приложение.

          4.8 • Рейтинг +11k

          Более 3 миллионов загрузок

          Бесплатно

          Сохранять

          Распечатать

          Редактировать

          Электрический ток

          СОДЕРЖАНИЕ :

          ОГЛАВЛЕНИЕ

            Электричество является формой энергии . Это явление, которое описывает поток заряженных частиц (особенно электронов) из одного места в другое. Все в мире состоит из атомов. Каждый атом состоит из ядра, окруженного отрицательно заряженными электронами. Ядро содержит частицы, называемые нейтронами (не имеющими заряда) и протонами (имеющими положительный заряд). Количество протонов и электронов одинаково в стабильном атоме, чтобы уравновесить общий нейтральный заряд.

            В проводниках (например, таких металлах, как медь или серебро) движение электронов, известное как свободных электронов , отвечает за перемещение заряда. Движущийся заряд — это то, что мы называем электрическим током .

            Явление электричества и его применения более подробно изучаются в области электротехники .

            Определение электрического тока

            Мы можем определить электрический ток как количество заряда, перемещающегося в течение определенного периода времени. Формула для расчета электрического тока и используемые единицы измерения следующие:

            • Базовой единицей СИ для электрического тока является ампер ( А ).
            • Ток (I) измеряется в амперах ( А ).
            • Q измеряется в кулонах ( C ).
            • Время (t) измеряется в секундах ( с ).
            • Заряд, ток и время связаны друг с другом соотношением Q = I ⋅ t.
            • Изменение заряда обозначается как ΔQ.
            • Точно так же изменение во времени обозначается как Δt.

            Еще один интересный момент заключается в том, что электрический ток создает магнитное поле, а магнитное поле также может создавать электрический ток.

            Вариант партии

            Когда два заряженных объекта соединяются с помощью проводника, через них протекает заряд, создавая ток. Ток течет, потому что разница в заряде вызывает разницу в напряжении.

            Рис. 1. Поток заряда в проводнике. Источник: StudySmarter.

            Таким образом, уравнение для протекания тока выглядит следующим образом:

            Условное протекание тока

            В цепи ток представляет собой поток электронов в цепи. Отрицательно заряженные электроны движутся от отрицательно заряженного вывода к положительно заряженному, следуя основному правилу: одноименные заряды отталкиваются, а противоположные притягиваются.

            Обычный ток описывается как поток положительного заряда от положительного вывода источника к его отрицательному полюсу. Это противоположно потоку электронов, как было сказано до того, как стало понятно направление тока.

            Рис. 2. Обычный поток в сравнении с потоком электронов. Источник: StudySmarter.

            Важно отметить, что ток имеет направление и величину, выраженную в амперах. Однако это не векторная величина.

            Как измерить ток

            Ток можно измерить с помощью устройства, называемого амперметром . Амперметры серии и всегда следует подключать к той части цепи, где вы хотите измерить ток, как показано на рисунке ниже.

            Это связано с тем, что ток должен проходить через амперметр, чтобы он мог считать значение. Идеальное внутреннее сопротивление амперметра равно нулю, чтобы на амперметр не попадало напряжение, поскольку оно может повлиять на цепь.

            Рис. 3. Устройство для измерения тока с помощью амперметра — StudySmarter Originals

            В: В каком из приведенных ниже вариантов через электрическую цепь проходит ток силой 8 мА?

            А. Когда заряд 4Кл проходит за 500с.

            B. Когда заряд 8C проходит за 100 с.

            С. Когда заряд 1С проходит за 8с.

            Раствор. Используя уравнение:

            I = 4/500 = 8 x 10-3 = 8 мА

            I = 8/100 = 80 x 10-3 = 80 мА

            I = 1/8 = 125 x 10- 3 = 125 мА

            Вариант А правильный: по цепи будет проходить ток 8 мА.

            Квантование заряда

            Заряд на носителях заряда квантуется , что можно определить следующим образом:

            Отдельный протон имеет положительный заряд, а отдельный электрон имеет отрицательный заряд. Этот положительный и отрицательный заряд имеет фиксированную минимальную величину и всегда кратен этой величине. 9-19 Кл. Мы можем представить заряд любой частицы как кратное этому.

            Расчет тока в проводнике с током

            В проводнике с током возникает ток, когда носители заряда свободно перемещаются. Заряд на носителях заряда может быть как положительным, так и отрицательным, и считается, что ток течет в одном направлении по проводнику. Ток в проводнике имеет несколько характеристик:

            • Носителями заряда в основном являются свободные электроны.
            • Хотя ток течет в определенном направлении в каждом проводнике, носители заряда движутся в противоположных направлениях со скоростью дрейфа v.
            • Первое изображение в Рис. 2 имеет положительные носители заряда. Здесь скорость дрейфа и носители заряда движутся в одном направлении. Второе изображение имеет отрицательные носители заряда, а скорость дрейфа и носители заряда движутся в противоположном направлении.
            • Скорость дрейфа носителей заряда — это средняя скорость, с которой они движутся по проводнику. 93).v — скорость дрейфа в м/с. q — заряд в кулонах. I — ток в амперах.

            Электрический ток — основные выводы

            • Электричество — это форма энергии. Это явление, которое описывает поток заряженных частиц (особенно электронов) из одного места в другое.
            • Базовая единица электрического тока в системе СИ – ампер (А) .
            • Обычный ток описывается как поток положительного заряда от положительного вывода элемента к отрицательному.
            • Заряд на носителях заряда квантуется .

            Часто задаваемые вопросы об электрическом токе

            Электрический ток измеряется в амперах (А) или амперах.

            Электрический ток определяется как скорость потока носителей заряда.

            Электрический ток всегда создает магнитное поле.

            Характеристики магнита используются для выработки электроэнергии. Электроны притягиваются и выталкиваются движущимися магнитными полями. Электроны в таких металлах, как медь и алюминий, рассеяны повсюду. Когда вы перемещаете магнит вокруг катушки с проволокой или катушку с проволокой вокруг магнита, электроны в проволоке выталкиваются, и создается электрический ток.

            Электрический ток является скалярной величиной. Любая физическая величина называется вектором, если она имеет величину, направление, а также подчиняется векторным законам сложения. Хотя электрический ток имеет величину и направление, он не подчиняется векторным законам сложения. Следовательно, электрический ток является скалярной величиной.

            Заключительный тест по электрическому току

            Вопрос

            Что такое электричество?

            Показать ответ

            Ответ

            Электричество – это форма энергии.

            Показать вопрос

            Вопрос

            Как проводится электричество?

            Показать ответ

            Ответ

            Путем перемещения заряда из одного места в другое.

            Показать вопрос

            Вопрос

            Какова единица измерения электрического тока в системе СИ?

            Показать ответ

            Ответ

            Единицей электрического тока в системе СИ является ампер (А) или амперы.

            Показать вопрос

            Вопрос

            Как математически связаны друг с другом заряд (Q), ток (I) и время (t)?

            Показать ответ

            Ответить

            Показать вопрос

            Вопрос

            Почему направление обычного тока и потока электронов отличается?

            Показать ответ

            Ответ

            Обычный ток противоположен потоку электронов, потому что он был описан до того, как стало понятно направление тока.

            Показать вопрос

            Вопрос

            Какой прибор мы используем для измерения электрического тока?

            Показать ответ

            Ответ

            Амперметр.

            Показать вопрос

            Вопрос

            Как следует подключать амперметры для измерения тока?

            Показать ответ

            Ответить

            Последовательно.

            Показать вопрос

            Вопрос

            Как квантуется заряд?

            Показать ответ

            Ответ

            Положительные и отрицательные заряды имеют фиксированную минимальную величину и всегда кратны этой величине.

            Показать вопрос

            Вопрос

            Какова дрейфовая скорость носителя заряда?

            Показать ответ

            Ответ

            Скорость дрейфа носителей заряда — это средняя скорость, с которой они движутся по проводнику.

            Показать вопрос

            Вопрос

             В каком из приведенных ниже вариантов по проводу проходит ток силой 2 мА?

            Показать ответ

            Ответ

            При переходе заряда 4С в 2000с.

            Показать вопрос

            Вопрос

            В каком из приведенных ниже вариантов по проводу проходит ток силой 3 мА?

            Показать ответ

            Ответ

            Когда заряд 3C проходит через 1000 с.

            Показать вопрос

            Вопрос

            Через провод проходит заряд в 10 кулонов, создавая ток в 1 ампер. Рассчитайте, сколько времени это займет, используя соотношение заряда, тока и времени.

            Показать ответ

            Ответ

            10 секунд.

            Показать вопрос

            Вопрос

            По проводу проходит заряд 4 Кл, создавая ток силой 2 ампера. Как долго это займет? Используйте зависимость заряда, тока и времени.

            Показать ответ

            Ответ

            2 секунды.

            Показать вопрос

            Вопрос

            Какой заряд имеют электроны, положительный или отрицательный?

            Показать ответ

            Ответ

             У них отрицательный заряд.

            Показать вопрос

            Вопрос

            Какой заряд имеют протоны, положительный или отрицательный?

            Показать ответ

            Ответ

            Они имеют положительный заряд.

            Показать вопрос

            Вопрос

            Какие два основных измерительных прибора в электрических цепях?

            Показать ответ

            Ответ

            Вольтметры и амперметры.

            Показать вопрос

            Вопрос

            Какое значение напряжения в точке заземления?

            Показать ответ

            Ответить

            Показать вопрос

            Вопрос

            Какой символ обозначает электрический ток?

            Показать ответ

            Ответ

            Показать вопрос

            Вопрос

            Каково основное назначение выключателя в электрических цепях?

            Показать ответ

            Ответить

            Чтобы остановить и запустить текущий поток.

            Показать вопрос

            Вопрос

            В идеале, какое сопротивление имеет диод, когда ток течет от его анода к катоду?

            Показать ответ

            Ответ

            Сопротивление равно нулю.

            Показать вопрос

            Вопрос

            В действительности, что происходит, если ток, протекающий через анод диода к катоду, не может достичь своего порогового уровня?

            Показать ответ

            Ответ

            Ток не проходит.

            Показать вопрос

            Вопрос

            Какова разница напряжений между выводами базы и эмиттера биполярного транзистора?

            Показать ответ

            Ответ

            0,7 вольта.

            Показать вопрос

            Вопрос

            Как называется положительный полюс компонента?

            Показать ответ

            Ответить

            Анод.

            Показать вопрос

            Вопрос

            Как называется отрицательный полюс компонента?

            Показать ответ

            Ответить

            Катод.

            Показать вопрос

            Вопрос

            В идеале какое значение сопротивления показывает диод, когда ток пытается течь от его катода к аноду?

            Показать ответ

            Ответ

            Значение сопротивления диода должно быть бесконечным.

            Показать вопрос

            Вопрос

            Какое уравнение для мощности в электричестве?

            Показать ответ

            Ответ

            P = V * I

            Показать вопрос

            Вопрос

            Какое уравнение для энергии в электричестве?

            Показать ответ

            Ответ

             E = P * t

            Показать вопрос

            Вопрос

            Что определяет направление петли при применении второго закона Кирхгофа?

            Показать ответ

            Ответ

            Вы можете выбрать направление петли.

            Показать вопрос

            Вопрос

            При применении второго закона Кирхгофа, если направление контура сначала проходит через положительный полюс, какое значение получит напряжение этого компонента?

            Показать ответ

            Ответ

            Положительный.

            Показать вопрос

            Вопрос

            Влияет ли направление петли на результат при применении второго закона Кирхгофа?

            Показать ответ

            Ответ

            Нет, направление цикла не влияет на результат.

            Показать вопрос

            Вопрос

            Что определяет полюса компонента?

            Показать ответ

            Ответ

            Способ, которым ток входит в компонент.

            Показать вопрос

            Вопрос

            При применении второго закона Кирхгофа, если направление контура сначала проходит через отрицательный полюс, какое значение получит напряжение этого компонента?

            Показать ответ

            Ответ

            Отрицательно.

            Показать вопрос

            Вопрос

            При вычислении алгебраической суммы токов в соединении, в каком порядке мы берем значения входных и выходных токов?

            Показать ответ

            Ответ

            Отрицательный и положительный.

            Показать вопрос

            Вопрос

            Второй закон Кирхгофа гласит, что в любом замкнутом контуре цепи алгебраическая сумма напряжений чему равна?

            Показать ответ

            Ответ

            Алгебраическая сумма напряжений равна нулю.

            Показать вопрос

            Вопрос

            Энергия сохраняется в электрических цепях. Правда или ложь?

            Показать ответ

            Ответ

            Верно. Энергия сохраняется в электрических цепях из-за закона сохранения энергии.

            Показать вопрос

            Вопрос

            Электрическая энергия получается из движения электронов. Правда или ложь?

            Показать ответ

            Ответ

             Верно, электрическая энергия получается из движения электронов.

            Показать вопрос

            Вопрос

            Что из следующего не относится к областям, в которых используются делители напряжения?

            Показать ответ

            Ответ

            Текущее измерение.

            Показать вопрос

            Вопрос

            Используются ли в мультиметрах делители напряжения?

            Показать ответ

            Ответ

            Да, в мультиметрах используются делители напряжения.

            Показать вопрос

            Вопрос

            Какой выходной коэффициент следует использовать для преобразования 10 вольт в 2,5 вольта?

            Показать ответ

            Ответ

            2,5/10

            Показать вопрос

            Вопрос

            Можно ли использовать делители напряжения для оптимизации электронного оборудования?

            Показать ответ

            Ответ

            Да, делители напряжения можно использовать для оптимизации электронного оборудования.

            Показать вопрос

            Вопрос

            Принимая во внимание закон Ома, если значение тока постоянно, какова связь между напряжением и сопротивлением?

            Показать ответ

            Ответ

            Они прямо пропорциональны.

            Показать вопрос

            Вопрос

            Можно ли использовать делители напряжения в качестве аттенюаторов сигналов?

            Показать ответ

            Ответ

            Да, делители напряжения можно использовать в качестве аттенюаторов сигнала.

            Показать вопрос

            Вопрос

            Когда вольтметрам нужны делители напряжения для измерения значения напряжения?

            Показать ответ

            Ответ

            Когда значение напряжения слишком велико.

            Показать вопрос

            Вопрос

            После того, как вы измерили величину напряжения, подаваемого через делитель напряжения, на какой коэффициент его следует умножить/разделить, чтобы получить правильный результат?

            Показать ответ

            Ответ

            Фактор, на который изначально было снижено входное напряжение.

            Показать вопрос

            Вопрос

            Может ли микроконтроллер считывать значение сопротивления датчика с помощью делителя напряжения?

            Показать ответ

            Ответ

            Да, микроконтроллер может считывать значение сопротивления датчика с помощью делителя напряжения.

            Показать вопрос

            Вопрос

            Для чего используются переключатели логических уровней?

            Показать ответ

            Ответ

            Для соединения двух логических схем.

            Показать вопрос

            Вопрос

            Что произойдет, если не использовать делитель напряжения при соединении двух логических схем, работающих при разных напряжениях?

            Показать ответ

            Ответ

            Цепи будут повреждены.

            Показать вопрос

            Вопрос

            Выходное напряжение делителя напряжения может быть найдено путем деления значения сопротивления выходного компонента на общее последовательное сопротивление и умножения на значение напряжения источника. Правда или ложь?

            Показать ответ

            Ответить

            Показать вопрос

            Вопрос

            Можно ли получить несколько выходных напряжений из схемы делителя напряжения с более чем двумя резисторами?

            Показать ответ

            Ответ

            Да, вы можете получить несколько выходных напряжений из схемы делителя напряжения с более чем двумя резисторами.

            Показать вопрос

            Вопрос

            Батарея имеет ЭДС 0,45 В. Ток, протекающий через батарею, составляет 0,07 А, а сопротивление нагрузки составляет 1,3 Ом. Найдите внутреннее сопротивление батареи.

            Показать ответ

            Ответить

            Показать вопрос

            Подробнее об электрическом токе

            Откройте для себя подходящий контент для ваших тем

            Не нужно обманывать, если у вас есть все необходимое для успеха! Упаковано в одно приложение!

            Учебный план

            Будьте идеально подготовлены вовремя с индивидуальным планом.

            Тесты

            Проверьте свои знания с помощью игровых тестов.

            Карточки

            Создавайте и находите карточки в рекордно короткие сроки.

            Заметки

            Создавайте красивые заметки быстрее, чем когда-либо прежде.

            Учебные наборы

            Все учебные материалы в одном месте.

            Документы

            Загружайте неограниченное количество документов и сохраняйте их в Интернете.

            Study Analytics

            Определите сильные и слабые стороны вашего исследования.

            Еженедельные цели

            Ставьте индивидуальные учебные цели и зарабатывайте баллы за их достижение.

            Умные напоминания

            Хватит откладывать на потом наши напоминания об учебе.

            Награды

            Зарабатывайте очки, открывайте значки и повышайте уровень во время учебы.

            Волшебный маркер

            Создавайте карточки в заметках полностью автоматически.

            Умное форматирование

            Создавайте самые красивые учебные материалы, используя наши шаблоны.

            Электрический ток — GeeksforGeeks

            Было время, когда не было никаких электронных устройств, таких как ноутбуки, мобильные телефоны, было даже время, когда у людей не было вентилятора, чтобы скрыть жару, света в доме, чтобы видеть в темноте , а в сегодняшнее время у Людей все это в руках, все это стало возможным благодаря одному изобретению — «Электричество». Электричество — это поток электрических зарядов, когда на концах провода имеется разность потенциалов. Не будет неправильным сказать, что Люди в основном полагаются на Электричество не только для легкой жизни, но и для того, чтобы стать образом жизни. От ракет до поездов, заводов и бытовой техники — все работает на электричестве.

            Что такое электрический ток?

            Поток электрических зарядов известен как электрический ток. Ранее, когда электроны еще не были открыты, говорили, что электрический ток — это поток положительных зарядов. Поток тока и сам ток создается за счет движения отрицательных зарядов (электронов), электроны текут от отрицательного вывода к положительному. клемма (потенциал от более низкого к более высокому), известная как поток электронов , в то время как ток течет от 9Положительная клемма 0420 к отрицательной клемме (от более высокого к более низкому потенциалу), которая также известна как обычный ток .

            Лучший способ представить ток, протекающий по цепи, — это представить движение жидкости внутри трубы от более высокой точки к более низкой, жидкость представляет собой ток, разница в высоте крайнего конца трубы представляет собой разность потенциалов, вызывающая протекание тока, если труба скручивается в какой-то момент, поток жидкости замедляется, что представляет собой сопротивление, оказываемое проводником, которое уменьшает ток.

            Формула электрического тока

            Электрический ток может быть представлен как скорость потока электрического заряда (q). Электрический заряд относится к основным единицам, таким как масса, длина и т. д. Единицей электрического заряда является кулон, поэтому единицей электрического тока становится кулон/сек.

            I = q/t

            Единица электрического тока

            Единицей электрического тока является ампер (А) или кулон в секунду.

            Электродвижущая сила 

            Обычно в проводнике присутствуют электроны, и они не являются стационарными, они продолжают двигаться в случайных направлениях, и из-за их случайности общее смещение всех электронов становится равным нулю и, следовательно, ток не вырабатывается. Чтобы произвести ток, требуется некоторая внешняя сила, чтобы выровнять электроны в одном направлении и заставить их двигаться в этом одном направлении, внешняя сила известна как электродвижущая сила , а также известна как ЭДС. Это не что иное, как напряжение, применяемое для получения тока.

            Свойства электрического тока

            • Ток возникает благодаря потоку электронов в цепи.
            • Электрический ток можно разделить на переменный и постоянный по своей природе, где постоянный ток — это постоянный ток, который течет только в одном направлении, постоянный ток используется в низковольтных устройствах, в самолетах и ​​т. д. Переменный ток известен как переменный ток, и он течет в обоих направлениях попеременно, переменный ток — это ток, который приходит в наши дома, и приборы работают от переменного тока.
            • Электрическим током в цепи можно управлять, вводя сопротивление в цепь.
            • Единицей электрического тока является Ампер (А). 1 ампер можно определить как поток заряда в 1 кулон за 1 секунду.
            • Электрический ток течет от более высокого потенциала к более низкому потенциалу в цепи (от положительной клеммы к отрицательной), также известной как обычное направление тока.

            Закон Ома

            Немецкий физик Георг Симон Ом заявил, что ток, протекающий по проводу, прямо пропорционален падению напряжения на нем. Согласно закону Ома, ток, протекающий по проводу, прямо пропорционален напряжению, приложенному к концам провода, при условии, что температура и проводимость остаются неизменными.

            После удаления знака пропорциональности вводится константа пропорциональности, известная как Сопротивление.

            В ∝ I

            В = RI

            В= Напряжение на концах проводника

            R= Сопротивление проводника

            I= Ток в проводнике.

            Эффект электрического тока

            Существуют различные эффекты, которые можно заметить из-за протекания электрического тока в проводе, например, когда ток проходит через резистор, резистор имеет свойство сопротивления, которое не позволяет весь ток проходит, но так как энергия не может быть ни создана, ни уничтожена, она преобразуется в тепловую энергию и высвобождается в виде тепла, этот эффект называется тепловым эффектом тока. Точно так же мы имеем магнитное и химическое действие электрического тока.

            Химический эффект электрического тока

            Когда электрический ток пропускают через токопроводящее вещество, раствор распадается на соответствующие ионы, и эффекты видны визуально. Основные заметные эффекты,

            • Цвет раствора может измениться.
            • Могут быть видны отложения металла на электродах.
            • Возможно образование пузырьков газа на электродах.

            Магнитное действие электрического тока

            Электрический ток есть не что иное, как движение электронов, и известно, что когда заряды неподвижны, они создают электрическое поле, а когда заряды движутся, они создают магнитное поле. Когда через провод пропускают ток и туда помещают металлический лист с иглой, игла будет отклоняться из-за наличия магнитного поля, создаваемого электрическим током. Одним из самых больших применений Магнитного эффекта электрического тока являются электромагниты, они образуются с помощью проходящего тока.

            Тепловой эффект электрического тока

            Когда ток течет по проводнику, тепловая энергия вырабатывается и выделяется из проводника, а величина эффекта зависит от сопротивления, оказываемого проводником. Если проводник должен иметь высокое сопротивление, это просто означает, что он не должен пропускать большую часть тока, но из-за сохранения энергии (энергия не может быть ни создана, ни уничтожена) ток, который не может пройти, преобразуется в тепло и явление известно как нагревательный эффект тока. Формула для тепловой энергии дается выражением,

            H = I 2 RT

            Где, H = выделяемая тепловая энергия

            I = ток, протекающий в проводнике

            R = сопротивление, оказываемое проводником

            T = время, в течение которого протекал ток в проводнике.

            Применения нагревательного эффекта тока включают электрические утюги, электрические нагреватели, лампы накаливания, электрические чайники и т. д.

            Примеры задач

            Вопрос 1. В проводнике протекает заряд 10 кулонов в течение 5 секунд, определите силу тока производится?

            Решение:

            Ток в цепи определяется выражением

            I = q/t

            I = 10/5 ампер схема.

            Вопрос 2: Ответьте на заданные вопросы,

            1. Какой материал лучше проводит электричество — железо или кремний?

            2. Какой проводник электричества лучше всего?

            Ответ: 

            1. Железо лучше проводит электричество, а кремний — полупроводник.

            2. Известно, что серебро является лучшим проводником электричества.

            Вопрос 3: В цепи, приведенной ниже, Найдите ток, протекающий через цепь.

            Решение:

            На приведенном рисунке видно, что есть два сопротивления, и они включены последовательно. Когда в цепи последовательно соединены два или более двух сопротивлений, общее сопротивление становится равным сумме отдельных сопротивлений, присутствующих в цепи.

            R = R 1 + R 2

            R = 2+ 2

            R = 4 ohms

            From Ohms Law, 

            V = IR

            I = V/R

            I= 20/ 4

            I= 5 Ампер

            Вопрос 4: Почему серебро не используется чаще в качестве проводника в повседневном использовании?

            Ответ:

            Серебро известно как лучший проводник электричества, но его все же не предпочитают использовать в повседневном использовании в качестве проводника из-за его недоступности и того факта, что серебро очень дорогое. Вместо этого используются легкодоступные проводники, такие как медь.

            Вопрос 5: Какая тепловая энергия будет произведена, если в течение 5 секунд по цепи протекает ток силой 2 ампера при общем сопротивлении цепи 4 Ом?

            Solution:

            The Heat energy produced is given by,

            H= I 2 RT

            H= (2) 2 ×4 × 5

            H= 16 × 5

            H= 80 Дж

            Следовательно, в контуре вырабатывается 80 Дж.


            Что такое электрический ток? Знайте интенсивность и ее виды – Окружные школы

            Содержание

            что такое электричество? Определение

            Электрический ток — это поток электрического заряда, который проходит через проводящий материал в течение определенного периода времени . Она выражается в Кл/с, кулонах в секунду в Международной системе единиц, и эта единица известна как Ампер (А).

            Чтобы появился электрический ток, электроны, наиболее удаленные от ядра атома материала, должны отделиться и свободно циркулировать между атомами указанного тела. Это явление также может иметь место с вариациями в природе, когда заряженные облака испускают струи электронов, которые циркулируют в воздухе и вызывают молнии.

            Для измерения электрического тока используется закон Ома ,   , который использует электрический ток, напряжение и сопротивление.

            Первые эксперименты с электричеством были проведены в 18 веке, и электрические заряды были получены только путем трения (статического) или индукции. Чтобы проверить постоянное движение электрического заряда, потребовалось до 1800 года, когда итальянский физик Алессандро Вольта изобрел электрическую батарею.

            Сила электрического тока

            Электрическая напряженность     — это заряд или электрический ток, который циркулирует через площадь в единицу времени, его обычно называют буквой I   (интенсивность), а единицей измерения обычно является  Ампер   (А) . Все электрические проводники должны выдерживать разную величину нагрузок, и чем выше нагрузка, тем больше сопротивление материала, из которого они изготовлены.

            Как производится электрический ток?

            Ток возникает в результате движения свободных зарядов (обычно электронов), находящихся внутри конкретного проводящего материала в электрической цепи. В замкнутой электрической цепи заряд электронов всегда движется от отрицательного полюса к положительному полюсу.

            Что означает этот ток?

               реальное направление электрического тока   всегда циркулирует электроны от отрицательного полюса (-) к положительному полюсу (+), однако обычное направление их циркуляции противоположно положительному полюсу к отрицательному полюсу.

            Это потому, что в свое время существование электронов не было известно, и научное сообщество решило применить этот тип вождения.

            Как измерить электрический ток?

            Чтобы измерить ток электричества в цепи, мы будем использовать закон Ома, который мы обсуждали ранее, формула выглядит следующим образом:

            Интенсивность = Напряжение / Сопротивление вольт, а сопротивление в омах.

            Иногда мы находим источники переменного тока  , которые постоянно меняют результат, в этом случае мы будем использовать такие инструменты, как мультиметр или амперметр   , который поможет нам правильно измерить силу тока.

            Эти мультиметры имеют 2 наконечника, которые позволяют измерять ток в серии .

            Виды электрического тока

            В зависимости от своей природы электрический ток может быть нескольких видов:

            • Постоянный ток (DC)

              Постоянный ток — вид электрического тока с непрерывным протеканием электрического заряда по проводнику между двумя точками разного потенциала и заряда, односторонней циркуляцией потока, не меняющейся от положительного полюса к отрицательному полюсу. Чтобы обозначить, что ток является непрерывным, необходимо, чтобы ток не менял направление, за истекшее время он всегда должен течь в одном и том же направлении. Интенсивность может варьироваться до тех пор, пока сохраняется одна и та же полярность.

            • Переменный ток (AC)

              Переменный ток — вид электрического тока, характеризующийся изменениями во времени как величины, так и направления через равные промежутки времени. Напряжение переменного сигнала изменяется между его максимумами и минимумами циклически, половина цикла положительна, а другая половина — отрицательна. Это означает, что ток циркулирует в обоих направлениях, в зависимости от того, положительный он или отрицательный. Этот цикл постоянно повторяется. Это тип энергии, который мы используем в наших домах для питания всех электроприборов с постоянной частотой 50 Гц. Он был разработан и приведен в действие Николой Теслой.

            • Однофазный ток

              Однофазный ток    достигается, когда берутся фаза трехфазного тока и нейтральный кабель. Это система, которая использует распределение, производство и потребление электроэнергии в одной фазе, поэтому напряжение всегда изменяется вместе.

              Чаще всего используется для электродвигателей, отопления или освещения.

            • Трехфазный ток

              Трехфазный ток представляет собой систему, состоящую из 3 переменных токов, которые отвечают за производство, распределение и потребление электроэнергии, их напряжения являются переменными и передаются проводящими системами, известными как R, S и T.

              Преимущество этого вида тока в том, что он более экономичен, чем другие, за счет экономии на трансформаторах и транспортных линиях, так как они представляют собой более тонкие провода, чем однофазный ток, что позволяет добиться большей производительности в двигателях.

            Воздействие электрического тока Когда электрическое сопротивление проводника очень мало, он выделяет тепло и свет.

            Электрический ток предлагает человечеству огромное количество практических преимуществ:

            Среди эффектов электрического тока обычно выделяют три:

            Статьи по теме

            Объяснение урока: Электрический ток | Nagwa

            В этом объяснителе мы узнаем, что такое электрический ток и как определить направление электрического тока в цепи.

            Электрический ток – это поток электрического заряда. Напомним, что электрический заряд исходит из разных частей атома, как показано ниже.

            Вместе положительно заряженные протоны, показанные розовым цветом, и нейтральные нейтроны, показаны зеленым цветом, составляют ядро. Отрицательно заряженные электроны, показанные на синие, находятся вне ядра.

            Электрический ток – это поток электрического заряда через электрическую проводник. Типичный электрический проводник представляет собой проволоку, изготовленную из металла, такого как медь, железо или серебро.

            При обсуждении потока электрического заряда движутся электроны через провод. Протоны и нейтроны не двигаются. Когда мы говорим об электрическом ток, мы имеем в виду поток электронов, движущихся в одном направлении вдоль тот же путь. На приведенной ниже диаграмме показана проволока, по которой движутся электроны.

            Электроны всегда находятся внутри провода, даже когда они не двигаются. Если электроны движутся, значит, через них течет электрический заряд. провод. Электрический ток — это поток электрического заряда по проводу.

            Если электроны находятся в проводе, но не двигаются, то электрический заряд не течет, как показано на схеме ниже.

            Электроны должны двигаться, чтобы возник электрический ток.

            Давайте рассмотрим пример.

            Пример 1. Определение того, какие части атома текут по проводу

            На рисунке показаны электроны и атомные ядра в срезе меди. провод. Синие кружки представляют электроны, а красные кружки представляют атомные ядра.

            1. Когда в проводе есть электрический ток, движутся ли электроны по проводу?
              1. Нет
              2. Да
            2. При наличии электрического тока в проводе атомные ядра двигаться по проводу?
              1. Да
              2. Нет

            Ответ

            Часть 1

            Когда в проводе есть электрический ток, электроны движутся. Если электроны не движутся, электрического тока нет.

            Ответ Б: да, электроны движутся по проводу.

            Часть 2

            При наличии электрического тока ядра атомов остаются неподвижными. Только электроны движутся при наличии электрического тока.

            Ответ на вторую часть Б: нет, атомные ядра не движутся провод.

            Электрический ток измеряется в Ампер, сокращенно А. Так 10 ампер может быть записывается как 10 А. Когда есть 0 А тока в цепи, электроны вообще не двигаются.

            Говоря конкретно о движении электронов в проводе, мы можем обратиться к к нему как поток электронов, также называемый потоком электронов.

            Электроны отрицательны, поэтому они движутся к положительному выводу клетки и от отрицательной клеммы, как показано на диаграмме ниже.

            Движение электронов создает электрический ток, поскольку никакое другое заряженная частица движется.

            Давайте рассмотрим пример вопроса.

            Пример 2. Определение направления потока электронов в цепи

            На схеме показана электрическая цепь, состоящая из элемента и лампочки.

            В каком направлении движутся электроны по цепи?

            1. По часовой стрелке
            2. Против часовой стрелки

            Ответ

            Направление потока электронов в цепи от отрицательного клемме клетки и к положительной клемме.

            Более короткая сторона элемента в верхней части этой цепи является его отрицательным Терминал. Более длинная сторона клетки является положительным полюсом. Электрон поток в этом контуре, таким образом, будет двигаться против часовой стрелки, как в контуре схема ниже.

            Правильный ответ B: против часовой стрелки.

            Хотя названия очень похожи, электронный ток и электрический ток не то же самое. Электронный ток — это поток электронов в проводнике, а электрический ток — это поток носителей заряда в проводе.

            Ранние ученые не знали, что электроны текут по цепи, когда электрический ток ток присутствовал. Когда эти ученые писали о потоке электрического заряд, они предположили, что поток электрического заряда происходит от положительно заряженных частицы.

            Это означает, что они измерили направление носителей электрического заряда от положительного полюса к отрицательному полюсу клетки. Такой электрический поток показан на диаграмме ниже.

            Соглашение, установленное этими учеными, до сих пор используется по умолчанию. направление электрического тока. Это текущее направление по умолчанию называется обычный ток.

            Направление обычного тока противоположно электронному току, так как он предполагает положительные носители заряда. На самом деле никаких положительных зарядов течь вообще; они чисто вымышленные. В проводе движутся только электроны. Показана диаграмма, сравнивающая обычный ток с электронным током. ниже.

            Электронный ток или поток электронов конкретно относится к потоку электроны. Электрический ток является более общим, поскольку он относится к потоку заряда перевозчиков, и по умолчанию принимает направление обычного Текущий.

            Давайте рассмотрим несколько примеров вопросов.

            Пример 3: Нахождение условного направления тока по потоку электронов

            На рисунке ниже показаны электроны и атомные ядра в разрезе медный провод. Синие кружки представляют электроны, а красные кружки представляют атомные ядра. В проводе протекает электрический ток, и электроны в нем движутся вправо.

            Каково направление условного тока в проводе?

            1. Вправо
            2. Влево

            Ответ

            Поток отрицательно заряженных электронов на диаграмме направлен вправо, но обычный ток предполагает, что носители заряда в токе положительный.

            Это означает, что обычное направление тока противоположно направление электронов. Это было бы слева, так как электроны текут вправо.

            Правильный ответ Б: налево.

            Пример 4. Определение условного направления тока в цепи

            На схеме показана электрическая цепь, содержащая элемент и лампочку.

            Каково направление условного тока в цепи?

            1. По часовой стрелке
            2. Против часовой стрелки

            Ответ

            Обычное направление тока в цепи предполагает положительные носители заряда. Это означает, что поток этих зарядов будет исходить от положительной клеммы ячейку к отрицательной клемме.

            Положительная клемма клетки, более длинная линия, направлена ​​вниз. Положительные заряды тогда должны были бы течь против часовой стрелки, что означает правильный ответ: B.

            Является ли носитель заряда фиктивным положительным зарядом в обычных ток или электрон в электронном токе, заряд должен течь, чтобы быть электрическим током.

            Неважно, сколько заряда присутствует, важно только то, что он движется. На приведенной ниже диаграмме показаны две цепи, одна из которых содержит намного больше электронов, чем Другой.

            Обе цепи имеют ток 0 А когда электроны не двигаются, независимо от того, сколько электронов есть.

            Электроны также не расходуются при уменьшении тока в цепи. Все электроны все еще присутствуют; они просто двигаются медленнее. Если ток упадет до 0, это означает, что электроны полностью перестали двигаться. На приведенной ниже диаграмме показано электроны движутся через лампочку, питая ее и заставляя ее загораться.

            Поскольку электроны движутся через лампочку, лампочка их не использует вверх или привести к их исчезновению. Движение электронов питает лампочку, поэтому, если электроны будут двигаться медленнее, лампочка будет более тусклой. Если бы электроны полностью перестанут двигаться, лампочка вообще не будет излучать свет.

            Давайте рассмотрим несколько примеров вопросов.

            Пример 5: Описание количества электронов в цепи после работы

            На схеме показана электрическая цепь, содержащая ячейку и лампочку. Эта схема установлена ​​на верстаке и оставлена ​​включенной на 1 час.

            В конце час, больше электронов, меньше электронов или такое же количество электронов в провода цепи, чем в начале час?

            1. В проводах больше электронов.
            2. В проводах меньше электронов.
            3. В проводах одинаковое количество электронов.

            Ответ

            Когда цепь включена, электроны движутся по цепи, питая лампочка, когда они проходят через нее.

            Электроны питают лампочку своим движением. В течение 1 час, некоторые электроны могли замедлиться из-за питания лампочки, но есть общее количество электронов в цепи остается тем же.

            В конце часа, после включения лампочки в ней остается такое же количество электронов. провода. Правильный ответ: C.

            Пример 6: Определение причины затемнения лампочки

            На схеме показана электрическая цепь, состоящая из элемента и лампочки. Эта схема установлена ​​на верстаке и оставлена ​​включенной на 1 час.

            В течение час, лампа постепенно тускнеет. Какое из следующих утверждений верно объясняет почему?

            1. Количество электронов в клетке со временем уменьшается, поэтому электроны, которые могут течь по цепи.
            2. Количество энергии в клетке со временем уменьшается, поэтому энергии становится меньше который может быть преобразован в свет лампочкой.

            Ответ

            Когда цепь включена, электроны движутся по цепи, питая лампочка, когда они проходят через нее.

            Когда лампочка тускнеет, это происходит не потому, что в ней меньше электронов. цепи, просто меньше электронов в движении, способных привести ее в действие. число электронов в цепи остается прежним.

            Поскольку энергия клетки медленно иссякает, она не может толкать столько электроны через провод и, таким образом, через лампочку.

alexxlab

Добавить комментарий

Ваш адрес email не будет опубликован. Обязательные поля помечены *